You are on page 1of 38

Copyright and Related Rights

Atty. Ferdinand Negre

I. INTRODUCTION COPYRIGHT vis-a-vis OTHER INTELLECTUAL PROPERTY 1. Kho v. CA (2002) Facts: Elidad Kho was engaged in the business of selling cosmetics under the name KEC Cosmetics. Kho filed an action for injunction with damages against Summervile General Merchandising for advertising and selling Khos alleged cream products under the CHIN CHUN SU brand as well as using the same containers thereby misleading the public and resulted in the decline of her business sales and income. Kho showed that she is the registered owner of the copyrights CHIN CHUN SU and Oval Facial Cream Container/Case by presenting Certificates of Copyright Registration. She also has patent rights on CHIN CHUN SU & DEVICE after purchasing it from the registered owner thereof in the Supplemental Register of the Phil. Pat. Off. Summerville, on the other hand, claimed to be the exclusive and authorized importer, repacker, and distributor of said products which were manufactured by a Taiwanese company who allowed Summerville to register its trade name Chin Chun Su Medicated Cream with the Phil. Patent Office & other agencies Issue: Is Kho entitled to the use of the trademark on Chin Chun Su and its container based on her copyright and patent over the same. Held: NO. Kho has no right to support her claim for the exclusive use of the subject trade name and its container. In order to be entitled to exclusively use the same in the sale of the beauty cream product, the user must sufficiently prove that she registered or used it before anybody else did. The petitioners copyright and patent registration of the name and container would not guarantee her the right to the exclusive use of the same for the reason that they are not appropriate subjects of the said intellectual rights. The name and container of a beauty cream product are proper subjects of a trademark inasmuch as the same falls squarely within its definition. Trademark, copyright and patents are different intellectual property rights that cannot be interchanged with one another. A trademark is any visible sign capable of distinguishing the goods (trademark) or services (service mark) of an enterprise and shall include a stamped or marked container of goods. In relation thereto, a trade name means the name or designation identifying or distinguishing an enterprise. Meanwhile, the scope of a copyright is confined to literary and artistic works which are original intellectual creations in the literary and artistic domain protected from the moment of their creation. Patentable inventions, on the other hand, refer to any technical solution of a problem in any field of human activity which is new, involves an inventive step, and is industrially applicable. PROTECTION of INTELLECTUAL PROPERTY and the CONSTITUTION 2. ABS-CBN v. PMSI (2009) Facts: ABS-CBN is engaged in television and radio broadcasting through wireless and satellite means while Philippine Multi-Media Systems Inc. (PMSI), the operator of Dream Broadcasting System provides direct-to-home (DTH) television via satellite to its subscribers all over the Philippines. PMSI was granted legislative franchise under RA 8630 to install, operate and maintain a nationwide DTH satellite service and is obligated under by NTC Memo Circ., Section 6.2 of which requires all cable television system operators operating within a certain community to carry the television signals of the authorized television broadcast stations (MUST CARRY RULE). ABS-CBN filed a complaint with IPO for violation of laws involving property rights. It alle ged that PMSIs unauthorized rebroadcasting of Channels 2 and 23 infringed on its broadcasting rights and copyright and that the NTC circular only covers cable television system operators and not DTH satellite television operators . Moreover, NTC Circular 4-08-88 violates Sec. 9 of Art. III of the Constitution because it allows the taking of property for public use without payment of just compensation. 1 Roco 2013

Copyright and Related Rights

Atty. Ferdinand Negre

PMSI argued that its rebroadcasting of Channels 2 and 23 is sanctioned by the said NTC Memo Circ.; that the must-carry rule under the Memorandum Circular is a valid exercise of police power. IPO and Court of Appeals ruled in favor of PMSI. Issue: (1) Whether PMSI infringed on ABS CBNs broadcasting rights and copyright (2) Whether PMSI is covered by the must carry rule Held: (1) NO. PMSI does not infringe on ABS-CBNs broadcasting rights under the IP Code as PMSI is not engaged in rebroadcasting of Channels 2 and 23. Broadcasting is defined as the transmission by wireless means for the public reception of sounds or of images or of representations thereof; such transmission by satellite is also broadcasting where the means for decrypting are provided to the public by the broadcasting organization or with its consent. Rebroadcasting, which is prohibited by the IP Code, is the simultaneous broadcasting by one broadcasting organization of the broadcast of another broadcasting organization. Also, broadcasting organizations are defined as entities that take the financial and editorial responsibility for the selection and arrangement of, and investment in, the transmitted content. Evidently, PMSI would not qualify as a broadcasting organization because it does not have the above responsibilities imposed upon broadcasting organizations, such as ABS-CBN. Sec 202.7 of the IP Code provides 2 instances wherein there is broadcasting: 1. The transmission by wireless means for the public reception of sounds or of images or of representations thereof. 2. The transmission by satellite for the public reception of sounds or of images or of representations thereof where the means for decrypting are provided to the public by the broadcasting organization or with its consent. ABS-CBN creates and transmits its own signals; PMSI merely carries such signals which the viewers receive in its unaltered form. PMSI does not produce, select, or determine the programs to be shown in Channels 2 and 23. Likewise, it does not pass itself off as the origin or author of such programs. Insofar as Channels 2 and 23 are concerned, PMSI merely retransmits the same in accordance with NTC Memo Circ. The retransmission of ABS-CBNs signals by PMSI which functions essentially as a cable television does not constitute rebroadcasting in violation of the formers intellectual property rights under the IP C ode. It must be noted that the law on copyright is not absolute. Under Sec 184, the use made of a work by or under the direction or control of the Govt, by the National Library, or by educational, scientific, or professional institutions where such use is in the public interest and is compatible with fair use shall not constitute copyright infringement. PMSI was granted a legislative franchise under Republic Act No. 8630, Section 4 of which similarly states that it shall provide adequate public service to enable the government, through the said broadcasting stations, to reach the population on important public issues; provide at all times sound and balanced programming; promote public participation such as in community programming; assist in the functions of public information and education. The Must-Carry Rule favors both broadcasting organizations and the public. It prevents cable television companies from excluding broadcasting organization especially in those places not reached by signal. Also, the rule prevents cable television companies from depriving viewers in far-flung areas the enjoyment of programs available to city viewers. The carriage of ABS-CBNs signals by virtue of the must-carry rule is under the direction and control of the government though the NTC which is vested with exclusive jurisdiction to supervise, regulate and control telecommunications and broadcast services/facilities in the Philippines. The imposition of the must-carry rule is within the NTCs power to promulgate rules and regulations, as public safety and interest may require, to encourage a larger and more effective use of communications, radio and television broadcasting facilities, and to maintain effective competition among private entities. (2) YES. DTH satellite tv operators is covered under the NTC Circular which requires all cable television system operators to carry the television signals of the authorized television broadcast stations. The Director-General of the IPO 2 Roco 2013

Copyright and Related Rights

Atty. Ferdinand Negre

and the CA correctly found that PMSIs services are similar to a cable television system because the services it renders fall under cable retransmission. Thus, PMSI, being a DTH Satellite TV operator is covered by the NTC Circular. II. SUBJECT MATTER of COPYRIGHT LAW A. Prerequisites for & Elements of Copyrightable Subject Matter ORIGINALITY 3. Burrow-Giles Lithographic Co. v. Sarony (1884) Facts: Sarony, a photographer, sued Burrow-Giles Lithographic Co. for allegedly violating his copyright in regard to a photograph, the title of which is Oscar Wilde, No. 18. Sarony had taken all the steps required by the act of congress to obtain copyright of this photograph. However, the lithographic company insists that a photograph, being a reproduction on paper of the exact features of some natural object or of some person, is not a writing of which the producer is the author; hence, not protected by copyright. Issues: (1) Whether or not there exists a constitutional right to protect photographs and negatives thereof by copyright. (2) W/N the words Copyright, 1882, by N. Sarony in the photographs, as a notice of the copyright of Napoleon Sarony are sufficient. Held: (1) Photographs are protected by copyright statutes. The Court held that Section 4952 of the Revised Statutes places photographs in the same class as things which may be copyrighted with books, maps, charts, dramatic or musical compositions, engravings, cuts, prints, paintings, drawings, statues, statuary, and models or designs intended to be perfected as works of the fine arts. Moreover, in an amending law, it states that he who shall invent and design, engrave, etch, or work, or from his own works shall cause to be designed and engraved, etched, or worked, any historical or other print or prints, shall have the same exclusive right for the term of 14 years from recording the title thereof as prescribed by law. Where a photographer so contrives the pose, costume, and expression of his subject as to produce an original and graceful effect, the picture is entitled to the benefit of the act providing for copyright of photographs. The only reason why photographs were not included in the extended list in the statute is, probably because they did not exist, as photography, as an art, was then unknown, and the scientific principle on which it rests, and the chemicals and machinery by which it is operated, have all been discovered long since that statute was enacted. Some argue that an engraving, a painting, a print embodies the intellectual conception of its author, in which there is novelty, invention, originality, and therefore comes within the protection of the law, while on the other hand, a photograph is the mere mechanical reproduction of the physical features or outlines of some object and involves no originality of thought or any novelty in the intellectual operation connected with its visible reproduction in shape of a picture. It is argued that the remainder of the process is merely mechanical, with no place for novelty, invention, or originality. It is simply the manual operation, by the use of these instruments and preparations, of transferring to the plate the visible representation of some existing object, the accuracy of this representation being its highest merit. The Court ruled that this may be true in regard to the ordinary production of a photograph, and that in such case a copyright is no protection. It should be noted that patents cannot, by law, be issued to the inventor until the novelty, the utility, and the actual discovery or invention by the claimant have been established by proof before the commissioner of patents; and when he has secured such a patent, and undertakes to obtain redress for a violation of his right in a court of law, the question of invention, of novelty, of originality is always open to examination. On the contrary, the copyright system has no such provision for previous examination by a proper tribunal as to the originality of the book, map, or other matter offered for copyright. A deposit of two copies of the article or work with the librarian of congress, with the name of the author and its title page, is all that is necessary to secure a copyright. It is therefore much more important that when the supposed author sues for a violation of his copyright, the existence of those facts of originality, of intellectual production, of thought, and conception on the part of the author should be proved than in the case of a patent right. We think this has been done by Sarony. 3 Roco 2013

Copyright and Related Rights

Atty. Ferdinand Negre

The finding of facts stated, in regard to the photograph in question that it is a useful, new, harmonious, characteristic, and graceful picture, and that plaintiff made the same entirely from his own original mental conception , to which he gave visible form by posing the said Oscar Wilde in front of the camera, selecting and arranging the costume, draperies, and other various accessories in said photograph, arranging the subject so as to present graceful outlines, arranging and disposing the light and shade, suggesting and evoking the desired expression, and from such disposition, arrangement, or representation, made entirely by plaintiff, he produced the picture in suit. These findings show that the photograph was original work of art, the product of Saronys intellectual invention, of which he is the author and for which he is secured the exclusive right to use, publish, and sell, as mandated by section 4952 of the Revised Statutes. Ultimately, the court held that the constitution is broad enough to cover an act authorizing copyright of photographs, so far as they are representatives of original intellectual conceptions of the author. (2) YES. The object of the statute/law is to give notice of the copyright to the public by placing upon each copy, in some visible shape, the name of the author, the existence of the claim of exclusive right, and the date at which this right was obtained. Notice is sufficiently given by the words Copyright, 1882, by N. Sarony found on each copy of the photograph. SOME NOTES: The Court clarified the proper definition of author to be he to whom anything owes its orign; originator; maker; one who completes a work of science or literature and writing is defined as the literary productions of those authors, and congress very properly has declared these to include all forms of writing, printing, engravings, etchings, etc., by which the ideas in the mind of the author are given visible expression. Furthermore, to support its decision, the court gave other definitions of the word Author: BRETT, M. R., said: An author is the person who effectively is as near as he can be the cause of the picture which is produced; that is, the person who has superintended the arrangement, who has actually formed the picture by putting the persons in position, and arranging the place where the people are to be---the man who is the effective cause of that. Lord Justice COTTON said: In my opinion, author involves originating, making, producing, as the inventive or master mind, the thing which is to be protected, whether it be a drawing, or a painting, or a photograph;' Lord Justice BOWEN said that: Photography is to be treated for the purposes of the act as an art, and the author is the man who really represents, creates, or gives effect to the idea, fancy, or imagination. 4. Bleistein v. Donaldson Lithographing Co. (DLC) (1903) Facts: Bleistein is an employee of Courier Lithographing Co. (CLC). CLC was hired by a certain Wallace to create chromolithographs to advertise his circus. When Wallace ran out, he asked DLC to copy in its reduced form 3 chromolithographs made by CLC. Before DLC published said copies, CLC copyrighted its creations. As a result, CLC, through Bleistein, sued DLC for copyright infringement. In its defense, DLC contended that the advertisements do not fall under copyright laws according it protection. Particularly, it does not fall under the clause of the US Constitution giving Congress power to promote the progress of science and useful arts, by securing for limited terms to authors and inventors the exclusive right to their respective works and discoveries for the reason that it is not considered as a USEFUL ART. Also, it does not fall under the provision of the Copyright Act which states that in the construction of this act, the words engraving, cut, and print shall be applied only to pictorial illustrations or works connected with the fine arts. Issue: Whether or not chromolithographs used for advertisements are protected by the copyright laws? Held: YES. The Constitution does not limit the USEFUL to that which satisfies immediate bodily needs. Regarding the provision in the Copyright Act, the Court held that these chromolithographs are pictorial illustrations. The word illustrations' does not mean that they must illus trate the text of a book, and that the etchings of Rembrandt or 4 Roco 2013

Copyright and Related Rights

Atty. Ferdinand Negre

Mller's engraving of the Madonna di San Sisto could not be protected today if any man were able to produce them. Again, the act, however construed, does not mean that ordinary posters are not good enough to be considered within its scope. Certainly works are not the less connected with the fine arts because their pictorial quality attracts the crowd, and therefore gives them a real use if use means to increase trade and to help to make money. A picture is nonetheless a picture, and none the less a subject of copyright, that it is used for an advertisement. And if pictures may be used to advertise soap, or the theatre, or monthly magazines, as they are, they may be used to advertise a circus. Printing and engraving, though not for a mechanical end, are not excluded from the useful arts, which congress is empowered by the constitution to promote by copyright laws. Justice Harlan, dissent: If a mere label simply designating or describing an article to which it is attached, and which has no value separated from the article, does not come within the constitutional clause upon the subject of copyright, it must follow that a pictorial illustration designed and useful only as an advertisement, and having no intrinsic value other than its function as an advertisement, must be equally without the obvious meaning of the Constitution. It must have some connection with the fine arts to give it intrinsic value. 5. Alfred Bell & Co. v. Catalda Fine Arts, Inc. (1951) Facts: Alfred sued Catalda for 8 separate copyright infringements for reproducing their mezzotint engravings of paintings of old masters which Alfred made and copyrighted. It was alleged that Catalda produced photo-lithographs of the mezzotint engravings made by Alfred. Alfred had exclusive licensing agreements with various museums for famous artworks from the 18th and 19th century. Having sole access to the said artworks, Alfred created mezzotint reproductions of the said paintings, one of which was Thomas Gainsboroughs Blue Boy which became the subject of this case, for sale. Catalda on the other hand wanted to reproduce the said artworks in their Fine Arts catalogue but having no access to the same, just opted to copy Alfreds copies thinking that since the originals were considered public domain artwork there would be no harm in copying a copy of it. In court, Catalda initially claimed that there was no copyright infringement since Alfred also copied the same from old paintings and there is no distinct difference from the originals with the mezzotint version that would warrant copyright protection. Then, when it was proven that Alfreds copyright was valid and thus there was indeed infringement on the part of Catalda, they put up the defense that Alfred was violating the anti-trust law since they were charging too much for its reproduction. So the court weighed the enforcement of the anti-trust law against prevention of infringement of copyright laws. In the case, defendants Catalda also asserts the requirement for patents, which is very strict, constitutionally governs copyright and they attack the constitutionality of the said sections of the Copyright Act as setting the standards too low for something to be copyrighted. One of their main contentions was on the use of the word original in granting copyright. Issue: (1) Whether the mezzotint of Alfred is copyrightable (2) Whether or not the low standard of procedure for copyrighting is constitutional Held: (1) YES. The fact that mezzotints were sufficiently distinguishable from said paintings warrant the granting of copyright protection. The mezzotint version had a richer color making it distinct from the original though everything else remained the same. Copyrights of the said mezzotints were valid and were infringed by Cataldas photo -lithograph reproduction. Despite statutory provisions declaring that no copyright shall exist in works in public domain, a The Constitutional and statutory provisions relating to copyrights are satisfied if author contributes something more than mere trivial variation in previous works and work may be copyrighted if there is something re cognizably the authors own and not mere copying of another. The Constitutional and statutory provisions relating to copyrights are satisfied if the author contributes 5 Roco 2013

Copyright and Related Rights

Atty. Ferdinand Negre

something more than mere trivial variation in previous works and the work may be copyrighted if there is something recognizable in the authors own and not mere copying of another (2) CONSTITUTIONAL. The imposition of far more exacting standards from inventors registering patents than in copyrights are valid mainly due to the fact that patents and copyrights are entirely distinct from each other. For patent, original refers to startling, novel, or unusual. Whereas, for copyright, it simply requires that the particular work owes its origin to the author. No large measure of novelty is necessary. The copyright of the book, if not pirated from other works, would be valid without regard to the novelty, or want of novelty, of its subject-matter. The novelty of the art described has nothing to do with the validity of the copyright. A copyright of a book merely secures to author exclusive right of printing and publishing book and does not give the author exclusive property of art described therein, but if he desires to acquire exclusive right to art recited, must attain patent therefore. Copyright is only a negative right to prevent the appropriation of the labors of an author by another . It could be shown that 2 similar works were in fact produced wholly independent of one another, one published it first would have no right to restrain the publication of the others independent and original work. A patentee has the right to prevent another from using his invention if it in fact infringes the formers patent , notwithstanding that the latters invention was the subject of independent investigation. 6. Muller & Co. v. New York Arrows Soccer Team Inc. (1986) Facts: John Muller & Company, Inc. (Muller) entered into a contract with Dr. David Schoenstadt, the owner of the New York Arrows Soccer Team, to create a logo to represent the team. A dispute over the fee for the service arose, and as a consequence, Muller sought to copyright the logo of the team. The Register of Copyrights refused to register the logo, not once, but twice, citing a lack of creativity. The logo consists of 4 angled lines which form an arrow and the word "Arrows" in cursive script below the arrow. The parties agreed that the copyrightability of the logo may be decided as a matter of law. Issue: Whether or not the logo created by Muller for the New York Arrows may be considered copyrightable. Held: NO. The US Courts ruled that there are two elements to be considered in determining whether a work is copyrightable, namely originality and minimal levels of creativity. In this case, the Court declared that the issue revolved around creativity, and not originality. It should be noted that Mullers position on the matter had a tendency to confuse the two with one another. If, as here, the creator seeks to register the item as a "work of art" or "pictorial, graphic or sculptural work, the work must embody some creative authorship in its delineation or form. There is no simple way to draw the line between "some creative authorship" and not enough creative authorship, and there are no cases involving "works" exactly like this one. 7. Meshwerks Inc v. Toyota Motor Sales USA (2008) Facts: For its advertising campaign, Toyota and its advertising agency Saatchi needed digital 3D models of its line of cars to appear in various media such as TVCs and websites. Meshwerks was hired to digitize the cars by mapping data points in a computerized grid to create a manipulatable wire frame of the car. Then, it fine -tuned the lines to make the digital models closely resemble the cars. Upon completion, Meshwerks registered said work to be copyrighted. After these models were made, Grace & Wild, Inc was hired to add necessary details in the models (color, texture, lighting, animation). These were then sent to Toyota to be used as advertisements. Toyota then used these designs in various ads across various media. According to Meshwerks, it should have only been for a single-use of its models as part of one Toyota TV commercial. Thereafter, Mershwerks sued Toyota , Saatchi & GW. Issue: Whether or not Toyota et al violated Meshwerks copyright over its 3D models. 6 Roco 2013

Copyright and Related Rights

Atty. Ferdinand Negre

Held: NO. To make a case for copyright infringement, plaintiff must show: (1) it owns a valid copyright; and (2) defendants copied constituent elements of the work copyrighted by plaintiff. While Meshwerks copyright registration creates a presumption of a valid copyright, this may be overcome by presenting evidence and legal arguments to establish that the works were not entitled to copyright protection. For a work to be copyrightable, it must contain a degree of originality since a copyright protection only extends to original works of authorship. Original works are (1) those independently created by the author; (2) not copied from other works; and (3) the work must possess some minimal degree of creativity. In this case, Meshwerks models are not so much independent creations as very good copies of Toyotas vehicles, as seen both in an objective assessment of the models as well as Meshwerks purpose and intent. Said models depict nothing more than UNADORNED Toyota vehicles. Meshwerks did not make any decisions regarding the lighting, shading, background, angle, etc. which would make depictions of things or facts in the world as a completely new expression subject to copyright protection. As such, having been stripped of any ingredient associated with an original expression, they have no copyrightable matter. One possesses no copyright interest in reproductions, when these reproductions do nothing more than accurately convey the underlying image. The fact that intensive, skillfull, and even creative labor is invested in the process of creating a product does not guarantee its copyrightability. 8. Mannion v. Coors Brewing Company (2006) Facts: Basketball magazine, SLAM, hired Mannion to photograph Kevin Garnett in relation to an article that the magazine intends to publish. Said article (Above the Clouds) appeared in its December 1999 issue and featured Mannions photo printed on 2-page spread. The Garnett Photo is a length portrait of Garnett against a backdrop of clouds with some blue sky shining through. The view is up and across the right side of Garnett's torso, so that he appears to be towering above earth. He wears avwhite T-shirt, white athletic pants, a black closefitting cap, and a large amount of bling bling, including several necklaces, a Rolex watch and bracelet on his left wrist, bracelets on his right wrist, rings on one finger of each hand, and earrings. His head is cocked, his eyes are closed, and his heavily-veined hands, nearly all of which are visible, rest over his lower abdomen, with the thumbs hooked on the waistband of the trousers. The light is from the viewer's left, so that Garnett's right shoulder is the brightest area of the photograph and his hands cast slight shadows on his trousers. In 2001, CHW Advertising proposed a design to Coors using a manipulated version of the Garnett Photo superimposed with Iced out and a can of Coors Light. CHWA obtained authorization from Mannions representative to use said photo in internal corporate merchandising catalog. This was chosen by Coors as its billboard photo. The Coors Billboard depicts, in black-and-white, the torso of a muscular black man, albeit a model other than Garnett, shot against a cloudy backdrop. The pose is similar to that in the Garnett Photograph, and the view also is up and across the left side of the torso. The model in the billboard photograph also wears a white T-shirt and white athletic pants. The model's bling-bling is prominently depicted. The light comes from the viewer's right, so that the left shoulder is the brightest part of the photograph, and the right arm and hand cast slight shadows on the trousers. Mannion noticed said Coors Billboard at 2 locations in LA. He then applied for registration of his copyright of the Garnett Photo in 2003 and brought an action for infringement on Feb 2004. The registration was completed in Feb 2004. Issue: Whether or not a photograph used in billboard ads for Coors infringes the photographers copyright . Held: YES. There can be no serious dispute that the Garnett Photograph is an original work

Roco 2013

Copyright and Related Rights

Atty. Ferdinand Negre

To prove infringement, a plaintiff with a valid copyright must demonstrate that: (1) the defendant has actually copied the plaintiffs work; and (2) the copying is illegal because a substantial similarity exists between the defendants work and the 8rotectable elements of plaintiffs. It is well-established that the sine qua non of copyright is originality and, accordingly, that copyright protection may extend only to those components of a work that are original to the author. Original in the copyright context means only that the work was independently created by the author (as opposed to copied from other works), and that it possesses at least some minimal degree of creativity. If a photographer arranges or otherwise creates the subject that his camera captures, he may have the right to prevent others from producing works that depict that subject. A photograph may be original in three respects. They are not mutually exclusive. (a) RENDITION: There may be originality which does not depend on creation of the scene or object to be photographed and which resides, instead, in such specialties as angle of shot, light and shade, exposure, effects achieved by means of filters, developing techniques etc. Thus, copyright protects not what is depicted but rather how it is depicted. (b) TIMING: A person may create a worthwhile photo by being at the right place at the right time. Copyright based on originality in timing is limited, just like in rendition, by the principle that copyright in a photograph ordinarily confers no rights over the subject matter. (c) CREATION OF SUBJECT: A photograph may be original to the extent that the photographer created the scene or subject to be photographed. Here, unlike in the previous 2, copyright extends to the subject matter. As to the Garnett Photo, the photograph does not result from slavishly copying another work. Moreover, its unusual angle and distinctive lighting support the fact that it is original in the rendition. Furthermore, Mannions in structions to KG that he wear simple and plain clothing and as much jewelry as possible gives it originality in the creation of the subject. Both photos share a similar composition and angle. The lighting is similar, and both use a cloudy sky as backdrop. The subjects are wearing similar clothing and similar jewelry arranged in a similar way. Coors, in other words, appears to have recreated much of the subject that Mannion had created and then, through imitation of angle and lighting, rendered it in a similar way. The similarities here thus relate to the Garnett Photographs originality in the rendition and the creation of the subject and therefore to its protected elements. Though there are differences (ie. black & white, mirror image, unidentified mans torso, different jewelry, T-shirt fit, Iced Out), these are more in the nature of changes rather than additions. It long has been the law that no plagiarist can excuse the wrong by showing how much of his work he did not pirate. 9. Ching Kian Chuan v. CA (2001) Facts: Ong imports vermicelli from a company based in Beijing and has a business under the firm name C.K.C. Trading. He repacks it in cellophane wrappers with a design of two-dragons and the TOWER trademark on the upper-most portion. Ong acquired a Certificate of Copyright Registration from the National Library on June 9, 1993 on the said design. She discovered that Tan repacked the vermicelli he imports from the same company but based in Qingdao in a nearly identical wrapper. Thus, Ong filed against Tan a complaint for infringement of copyright with damages and prayer for TRO or writ of preliminary injunction. Ong alleged that he was the holder of a Cert. of Copyright Registration over the cellophane wrapper with the two-dragon design and that Tan used an identical wrapper. Tan alleged that Ong was not entitled to an injunction because Ong did not have a clear right over the use of the trademark Pagoda and Lungkow vermicelli as these were registered in the name of Ceroilfood Shandong, based in Qingdao, China. Tan averred that he was the exclusive distributor in the Philippines of the Pagoda and Lungkow vermicelli and was solely authorized to use said trademark. He added that Ong merely copied the two-dragon design from Ceroilfood Shandong which had the Certificates of Registration issued by different countries. Tan concluded that Ongs Certificate of Copyright Registration was not valid for lack of originality. CA granted the injunction was granted against Ong and held that Ongs wrapper is a copy of Ceroilfood Shandongs wrapper. 8 Roco 2013

Copyright and Related Rights

Atty. Ferdinand Negre

Issue: Whether or not Ong has a better right over Tan. Held: NO. A person to be entitled to a copyright must be the original creator of the work. He must have created it by his own skill, labor and judgment without directly copying or evasively imitating the work of another. A grant of preliminary injunction depends chiefly on the extent of doubt on the validity of the copyright, existence of infringement, and the damages sustained by such infringement. In the courts view, the copies of the certificates of copyright registered in the name of Ceroilfood Shandong sufficiently raise reasonable doubt. With such a doubt, the preliminary injunction is unavailing. As for Ong, the Court noted that for him to be entitled to an injunctive writ, he must show the existence of a clear and unmistakable right and an urgent and paramount necessity for the writ to prevent serious damage. The court found that Ongs right has not been clearly and unmistakably demonstrated. That right is wh at is in dispute and has yet to be determined. 10. Sambar v. Levi Strauss (2002) Facts: CVS Garment Enterprises (CVSGE) used an arcuate stitching design on the back pockets of Europress jeans. As such, Levis demanded from CSVGE to desist from using said desi gn in the jeans they were selling. They also advertised via Manila Bulletin. Counsel of CVSGE replied saying that the design in Europress jeans was different from that of Levis. It was also added that CVGSE had a copyright on the design it was using. Thus, Levis sued Sambar and CVSGE for infringement and unfair competition. Levis indicated in their complaint that they are an internationally known clothing manufacturer and owns the arcuate design trademark registered under US Trademark Registration dated Nov. 16, 1943. Also, it was registered in the Principal Register of trademarks with the Phi Patent Office dated Oct. 8, 1973. For CVSGE, while admitting that it was still manufacturing Europress jeans with the questioned arcuate design, it denied that there was infringement because the public would not be confused on the ownership of said trademark. Also, it claimed that it had its own arcuate design as evidenced by their Copyright Registration. Lower courts and the CA ruled that there was infringement, awarded damages to Levis and ordered the Director of the National Library to cancel the Copyright Reg issued in Samars name. Issue: Whether or not Sambar and CVSGE are liable for copyright infringement? Held: YES. Other than the fact that the issues raised are factual, and thus, not within the ambit of the SC, the mere fact that Sambar had a copyright over the design and allowed the use of it by CSVGE makes him liable. Also, to be entitled to copyright, the thing being copyrighted must be original, created by the author through his own skill, labor, and judgment, without directly copying or evasively imitating the work of another. Regarding damages, the fact that there was infringement means they suffered losses for which they are entitled to moderate damages. CREATIVITY 11. Feist Publications, Inc. v. Rural Telephone Service Co. (1991) Facts: Rural Telephone Service Company, Inc. (Rural) is a public utility providing telephone services in several Kansas communities. State regulations mandates them to annually publish an undated telephone directory white & yellow pages. The white pages alphabetically listed the subscribers names while the yellow pages though also alphabetically listed, consisted of classified advertisements from which they earn revenue. Thus, they obtained information from their subscribers, published a directory and copyrighted the same. This was given to subscribers free of charge. Feist Publications, Inc. (Feist) is a publishing company specializing in area-wide telephone directories for much larger areas. They compete with Rural for yellow page advertising. Feist is not a telephone provider, unlike Rural who 9 Roco 2013

Copyright and Related Rights

Atty. Ferdinand Negre

monopolizes the area. So, they approach all 11 telephone providers and offer to pay for their white pages listing so they can compile and publish the same. Rural refused to license its white pages listing to Feist for the project directory covering 11 different service areas in 15 countries with 46,878 white pages listings compared to Rurals 7,700 listings. As a result, Feist extracted the listing from Rurals directory without consent. It was later found out by the district court that this was Feists attempt to extend its mo nopoly in the yellow pages advertising as well. Most of the contents of the listing from Rural were altered since those not covered in Feists geographical area were deleted, they also added specific street addresses not in Rurals list, but several were identical including 4 fictitious entries which Rural specifically included to detect copying . Rural sued for copyright infringement. The District Court granted summary judgment in favor of Rural holding the directories as copyrightable, which the CA affirmed. Issue: Whether the copyright in Rurals directory protects the names, towns, and telephone numbers copied by Feist Held: NO! The facts involved from Rurals white pages were not proper subject matter of copyright registration. Feists use of them does not constitute infringement. Rural concedes that No author may copyright his ideas or the facts he narrates. But they claim that the compilation of facts is copyrightable as expressly mentioned in Copyright Act of 1976 and 1909. The US Constitution mandates originality as a prerequisite for copyright protection independent creation plus a modicum of creativity. Since the facts do not owe their origin to an act of authorship, they are not original, and thus, not copyrightable. Compilation of facts may possess the requisite originality if the author chooses which facts to include , order of placing them, manner of arrangement, and the like. Copyright protection only extends to these components original to the author and not to the facts themselves. A compilation though copyrightable receives a limited protection and does not extend to the facts. The sweat of the brow or industrious collection test which gives credit to the effort of the compiler so as to extend protection beyond the selection and arrangement to the facts themselves as used by lower courts misconstrued the Copyright Act of 1909 and 1976. No one may copyright facts or ideas. Factual compilations (such as a telephone directory) may possess required originality to qualify for copyright purposes; choices as to selection and arrangement, if independently made by compiler and entailing minimal degree of creativity, are sufficiently original to be subject to protection under copyright laws. Work is original to author and qualifies for copyright protection if said work is independently created by author and possesses some minimal degree of creativity. Both elements must be present to be included under the protection of the law. A directory containing absolutely no protectable written expression, only facts, meets constitutional minimum for copyright protection if it features original selection and arrangement. Rurals white pages do not meet the constitutional or statutory requirements for copyright protection. They have a valid copyright in the directory as a whole for the forward texts it contains and original material in the yellow pages, but nothing in the white pages. Their selection of listing-subscribers names, towns, telephone numbers lacks the necessary creativity since they are just listed alphabetically. There was also obviously no selection done since those were the exact information mandated by the state regulation to be contained in the directory. B. When Copyright Vests 12. Santos v. McCullough Printing (1964) Facts: Mauro Malang Santos (Santos) was commissioned by Ambassador Neri to create a Christmas card design for 1959. The design consisted of a holiday scene in the province, complete with token carabao, mango tree, and parol. Santos penned the work with his pen name Malang, visible on the design. It should be noted that Santos never had the design copyrighted. Ambassador Neri distributed the card to over 800 people. 10 Roco 2013

Copyright and Related Rights

Atty. Ferdinand Negre

In 1960, the McCullough Printing Co. (McCullough) displayed the same design in its album of Christmas cards which were offered for sale. McCullough received order for over 900 copies of the design. Santos claims damages for the unauthorized use of his design by McCullough, as he claimed that the said design was made specifically for Ambassador Neri and was intended for limited use only. Issue: Whether or not copyright is vested in Santos, as the undisputed creator of the design. Held: NO. The law states that, in order for an intellectual creation to be protected, it must be registered within 30 days from its creation if made in Manila, or within 60 is made elsewhere . The design of Santos was never registered for copyright. There being no registration, the protection of the laws cannot be afforded to Santos. In addition, the fact that Santos signed the design with his pen name works against him. While such a signature indicates that he is the creator of the work, it does not bar McCullough from using the design. In fact, it protects McCullough, as it gives credit to Santos, and therefore does not constitute piracy nor plagiarism. As to the contention of Santos that his work was intended to be limited in nature , the Court held that the facts work against him. First, there was no publication whatsoever that the design was meant to be limited. If such limited nature was indeed present, Ambassador Neri should be the complainant, as he was the person who commissioned the design with the intention to be limited and one of a kind. Second, the design was distributed to over 800 people. Such a mass distribution negated the limited edition nature of the design, as explained by the Court : When the purpose is a limited publication, but the effect is general publication, irrevocable rights thereupon become vested in the public, in consequence of which enforcement of the restriction becomes impossible. When Ambassador Neri distributed 800 copies of the design in controversy, the plaintiff lost control of his design and the necessary implication was that there had been a general publication, there having been no showing of a clear indication that a limited publication was intended. The author of a literary composition has a right to the first publication thereof. He has a right to determine whether it shall be published at all, and if published, when, where, by whom, and in what form. This exclusive right is confined to the first publication. When once published, it is dedicated to the public, and the author loses the exclusive right to control subsequent publication by others, unless the work is placed under the protection of the copyright law. There is no disputing that Santos is the owner of the design, as he created it, and at the time of creation, he has absolute dominion over it. However, once it had undergone general publication, he lost the right to control subsequent publication over it. Unfortunately for Santos, since there was no registration for copyright whatsoever in reference to the design. Therefore, he cannot be protected by the laws and the copyright was never vested to him. 13. Filipino Society of Composers v. Benjamin Tan (1987) Facts: Petitioners is an association of authors, composers, and publishers. It owns musical compositions like Dahil Sa Iyo, Sapagkat Ikaw Ay Akin, Sapagkat Kami Ay Tao Lamang, and The Nearness of You. All of which have been copyrighted. On the other hand, Tan operates Alex Soda Foundation and Restaurant. In 1967, a band was hired to amuse customers therein and played the above compositions without any license or permission from Petitioners. Accordingly, they demanded from Tan to pay license fees for playing said compositions in the restaurant. The later refused. Thus, Petitioner filed a complaint for infringement. Issue: (1) Whether or not the playing/singing of copyrighted musical compositions inside a restaurant constitute a public performance for profit within the meaning and contemplation of Copyright Law (Act 3134) (2) Assuming that it was indeed a public performance for profit, whether or not restaurant can be held liable. Held: (1) YES. It is beyond question that the playing and singing of the combo Tans restaurant constituted performance for profit contemplated by the Copyright Law. The music provided is for the purpose of entertaining the customers to make the establishment more desirable. 11 Roco 2013

Copyright and Related Rights

Atty. Ferdinand Negre

(2) NO. The composers of the musical compositions waived their right in favor of the general public when they allowed their intellectual creations to become property of the public domain before applying for the corresponding copyrights forthe same. Pursuant to the law, an intellectual creation should be copyrighted 30 days after its publication, if made in Manila, or within 60 days if made elsewhere, failure of which renders such creation public property. It was shown that the compositions became popular in radios, juke boxes, etc long before they were registered. Thus, these compositions had long become public property and beyond the protection of the Copyright Law. 14. Unilever Phils. v. CA (2006) Facts: As early as 1982, a P&G Italy used a key visual in the advertisement of its laundry detergent and bleaching products. This key visual known as the double-tug or tac-tac demonstration shows the fabric being held by both hands and stretched sideways. The tac-tac was conceptualized for P&G by the advertising agency Milano and Gray in 1982. The tac-tac was used in the same year in an advertisement to demonstrate the effect on fabrics of one of P&GP's products, a liquid bleach called Ace. Since then, P&G has used the tac-tac key visual in the advertisement of its products. In fact, in 1986, in Italy, the tactac key visual was used in the television commercial for Ace entitled Kite. P&G has used the same distinctive tactac key visual to local consumers in the Philippines. Unilever, on July 1993 aired a TVC of its Breeze Powerwhite laundry product called Porky. The said TVC included a stretching visual presentation and sound effects almost identical or substantially similar to P&GP's tac-tac key visual. On July 1994, P&GP aired in the Philippines, the same Kite television advertisement it used in Italy in 1986, merely dubbing the Italian language with Filipino for the same product Ace bleaching liquid which P&GP now markets in Philippines. Unilever filed a Complaint with the Advertising Board of the Philippines to prevent P&GP from airing the Kite television advertisement. A writ of preliminary injunction was issued against Unilever by the CA, enjoining it from using and airing TVCs for its laundry products claimed to be identical or similar to P&Gs key visual. Issue: Whether the issuance of preliminary injunction against Unilever was proper Held: NO. Unilever does not deny that the questioned TV advertisements are substantially similar to P&GP's double tug or tac-tac key visual. However, it submits that P&GP is not entitled to the relief demanded for the reason that Unilever has Certificates of Copyright Registration for its advertisements while P&GP has none with respect to its double-tug or tac-tac key visual. In other words, it is Unilevers contention that P&GP is not entitled to any protection because it has not registered with the National Library the very TV commercials which it claims have been infringed by petitioner. Contrary to Unilevers contention, the intellectual creator's exercise and enjoyment of copyright for his work and the protection given by law to him is not contingent or dependent on any formality or registration. P&GP is entitled to the injunctive relief prayed for in its Complaint. Section 2 of PD 49 stipulates that the copyright for a work or intellectual creation subsists from the moment of its creation. Accordingly, the creator acquires copyright for his work right upon its creation. C. Scope of Copyright THE IDEA/EXPRESSION DICHOTOMY 15. Baker v. Selden (1879) Facts: Charles Selden obtained a copyright of a book, entitled Selden's Condensed Ledger, or Bookkeeping Simplified, the object of which was to exhibit and explain a peculiar system of bookkeeping. He took the copyright of several other books, containing additions to and improvements upon the said system.

12

Roco 2013

Copyright and Related Rights

Atty. Ferdinand Negre

Complaint was filed against Baker for an alleged infringement of these copyrights. Baker denied that Selden was the author or designer of the books, and denied the infringement charged, and contends that the matter alleged to be infringed is not a lawful subject of copyright. The book or series of books of which the complainant claims the copyright consists of an introductory essay explaining the system of bookkeeping referred to, to which are annexed certain forms or banks, consisting of ruled lines, and headings, illustrating the system and showing how it is to be used and carried out in practice. The system effects the same results as bookkeeping by double entry; but, by a peculiar arrangement of columns and headings, presents the entire operation, of a day, a week, or a month, on a single page, or on two pages facing each other, in an account book. Baker uses a similar plan so far as results are concerned; but makes a different arrangement of the columns, and uses different headings. Baker used the same system as that which is explained and illustrated in Selden's books. Selden contended that the ruled lines and headings, given to illustrate the system, are a part of the book, and, as such, are secured by the copyright; and that no one can make or use similar ruled lines and headings, or ruled lines and headings made and arranged on substantially the same system, without violating the copyright. Issue: Whether the exclusive property in a system of bookkeeping can be claimed, under the law or copyright, by means of a book in which that system is explained. Held: NO. The system of Selden was not entitled to copyright protection. Blank account-books are not the subject of copyright; and that the mere copyright of Selden's book did not confer upon him the exclusive right to make and use account-books, ruled and arranged as designated by him and described and illustrated in said book. As an author, Selden explained the system in a particular way. It may be conceded that Baker makes and uses accountbooks arranged on substantially the same system; but the proof fails to show that he violated the copyright of Selden's book, regarding the latter merely as an explanatory work; or that he has infringed Selden's right in any way, unless the latter became entitled to an exclusive right in the system. There is a clear distinction between the book, as such, and the art which it is intended to illustrate. The copyright of the book, if not pirated from other works, would be valid without regard to the novelty, or want of novelty, of its subject matter. The novelty of the art or thing described or explained has nothing to do with the validity of the copyright. To give to the author of the book an exclusive property in the art described therein, when no examination of its novelty has ever been officially made, would be a surprise and a fraud upon the public. That is the province of letters patent, not of copyright. The claim to an invention or discovery of an art or manufacture must be subjected to the examination of the Patent Office before an exclusive right therein can be obtained; and it can only be secured by a patent from the government. Take the case of medicines. Certain mixtures are found to be of great value in the healing art. If the discoverer writes and publishes a book on the subject (as regular physicians generally do), he gains no exclusive right to the manufacture and sale of the medicine; he gives that to the public. If he desires to acquire such exclusive right, he must obtain a patent for the mixture as a new art, manufacture, or composition of matter. He may copyright his book, if he pleases; but that only secures to him the exclusive right of printing and publishing his book. By publishing the book, without getting a patent for the art, the latter is given to the public. The fact that the art described in the book by illustrations of lines and figures, which are reproduced in practice in the application of the art, makes no difference. The very object of publishing a book on science or the useful arts is to communicate to the world the useful knowledge which it contains. But this object would be frustrated if the knowledge could not be used without incurring the guilt of piracy of the book. Of course, these observations are not intended to apply to ornamental designs, or pictorial illustrations 13 Roco 2013

Copyright and Related Rights

Atty. Ferdinand Negre

addressed to the taste. Of these it may be said, that their form is their essence, and their object, the production of pleasure in their contemplation. This is their final end. On the other hand, the teachings of science and the rules and methods of useful art have their final end in application and use. This application and use are what the public derives from the publication of a book, which teaches them. Charles Selden, by his books, explained and described a peculiar system of book-keeping, and illustrated his method by means of ruled lines and blank columns, with proper headings on a page, or on successive pages. Now, whilst no one has a right to print or publish his book, or any material part thereof, as a book intended to convey instruction in the art, any person may practice and use the art itself which he has described and illustrated therein. The use of the art is a totally different thing from a publication of the book explaining it. The copyright of a book on bookkeeping cannot secure the exclusive right to make, sell, and use account-books prepared upon the plan set forth in such book. The system was not patented, and is open and free to the use of the public. And, of course, in using the art, the ruled lines and headings of accounts must necessarily be used as incident to it. The description of the art in a book, though entitled to the benefit of copyright, lays no foundation for an exclusive claim to the art itself. The object of the one (publication of book) is explanation; the object of the other (art/system) is use. The former may be secured by copyright. The latter can only be secured, if it can be secured at all, by letters-patent. 16. A.A. Hoehling v. Universal City Studios (1980) Facts: Hoehling published a book entitled Who Destroyed the Hindenburg? The book was based on the Hindenberg, a luxury airship (zeppelin) created by Germany during Hitlers time. On May 6, 1937, the Hindenburg hovered over Lakehurst, New Jersey from its voyage from Germany. At 7:25 pm, it exploded into flames and disintegrated in 35 seconds. 36 people were killed. As a result, investigations from American and German authorities ensued but they could not ascertain the definite cause of the tragedy. Throughout the entire investigation, the authorities never ruled out sabotage as the cause for said tragedy. The incident sparked a lot of journalists and columnists to write about the incident using the reports from the investigation as basis. Hoehling was no different. He studied investigative reports, consulted previously published articles and books, and conducted interviews with survivors of the crash as well as others who possessed information about the Hindenburg. His book is presented as a factual account, written in an objective, reportorial style. In the final chapter, spanning 11 pages, Hoehling suggests that all proffered explanations of the explosion, save deliberate destruction, are unconvincing. He concludes that the most likely saboteur is one Eric Spehl, a rigger on the Hindenburg crew who was killed at Lakehurst. Hoehling opined that Spehl destroyed the airship by placing a bomb its gas cell to please his ladyfriend who was dedicated to disproving Nazi invincibility. Ten years later, Mooney published his book The Hindenberg. His endeavor might be characterized as more literary than historical in its attempt to weave a number of symbolic themes through the actual events surrounding the tragedy. The destruction is depicted as the ultimate triumph of nature over technology, as Spehl plants the bomb that ignites the hydrogen. Developing this theme from the outset, Mooney begins with an extended review of man's efforts to defy nature through flight, focusing on the evolution of the zeppelin. This story culminates in the construction of the Hindenburg, and the Nazis' claims of indestructibility. Mooney then traces the fateful voyage, advising the reader almost immediately of Spehl's scheme. The book concludes with the airship's explosion. Mooney acknowledges, in this case, that he consulted Hoehling's book, and that he relied on it for some details. He asserts that he first discovered the Spehl-as-saboteur theory when he read Titler's Wings of Mystery. Indeed, Titler concludes that Spehl was the saboteur, for essentially the reasons stated by Hoehling. He also claims to have studied the complete National Archives and New York Times files concerning the Hindenburg, as well as all previously published material. Moreover, he traveled to Germany, visited Spehl's birthplace, and conducted a number of interviews with survivors. After Mooney prepared an outline of his anticipated book, his publisher succeeded in negotiations to sell the motion picture rights to appellee Universal City Studios. A certain Gidding was asked to write the screenplay. Said screenplay 14 Roco 2013

Copyright and Related Rights

Atty. Ferdinand Negre

included other fictional characters and subplots. In the film, which was released in late 1975, a rigger named Boerth, who has an anti-Nazi ladyfriend, plans to destroy the airship in an effort to embarrass the Nazis. Upon learning of Universal's plans to release the film, Hoehling instituted this action against Universal for copyright infringement and common law unfair competition. He also impleaded Mooney. Hoehling's principal claim is that both Mooney and Universal copied the essential plot of his book i. e., Eric Spehl, influenced by his girlfriend, sabotaged the Hindenburg. Issue: Whether or not Universal is liable to Hoehling for copyright infringement. Held: NO. To avoid a chilling effect on authors who contemplate tackling an historical issue or event, broad latitude must be granted to subsequent authors who make use of historical subject matter, including theories or plots. It is undisputed that Hoehling has a valid copyright in his book. To prove infringement, however, he must demonstrate that Universal copied his work and that they improperly appropriated his expression. Ordinarily, wrongful appropriation is shown by proving a substantial similarity of copyrightable expression. While Hoehling's Spehl destroys the airship to please his communist girlfriend, Mooney's character is motivated by an aversion to the technological age. Universal's Boerth, on the other hand, is a fervent anti-fascist who enlists the support of a Luftwaffe colonel who, in turn, unsuccessfully attempts to defuse the bomb at the crucial moment. Hoehling, correctly rejoins that while ideas themselves are not subject to copyright, his expression of his idea is copyrightable. However, the idea at issue is an interpretation of an historical event , and based on case law, such interpretations are not copyrightable as a matter of law . Although the plots of the 2 works were necessarily similar, there could be no infringement because of the public benefit in encouraging the development of historical and biographical works and their public distribution. The hypothesis that Eric Spehl destroyed the Hindenburg is based entirely on the interpretation of historical facts. Such an historical interpretation, whether or not it originated with Mr. Hoehling, is not protected by his copyright and can be freely used by subsequent authors. Factual information is part of the public domain. The similar elements in Hoehlings, Mooney, & Universals work are merely scenes a faire, that is, incidents, characters or settings which are as a practical matter indispensable, or at least standard, in the treatment of a given topic. Because it is virtually impossible to write about a particular historical era or fictional theme without employing certain stock or standard literary devices, we have held that scenes a faire ar e not copyrightable as a matter of law. A verbatim reproduction of another work, of course, even in the realm of nonfiction, is actionable as copyright infringement. Courts should assure themselves that the works before them are not virtually identical. In this case, it is clear that all three authors relate the story of the Hindenburg differently. In works devoted to historical subjects, it is our view that a second author may make significant use of prior work, so long as he does not bodily appropriate the expression of another. 17. American Dental Association v. Delta Dental (1997) CASE BRIEF: A system of identification codes for dental procedures is copyrightable. ADA developed a system that assigned a numerical code to each dental procedure. Under the system, each procedure was assigned a numerical code, a short description, and a long description. The court found that Delta had infringed ADAs copyright to the system by copying the codes and the descriptions. According to the court, ADAs system exhibited originality because dental procedures could be classified by complexity, or by the tools necessary to perform them, or in any of a dozen different ways.

15

Roco 2013

Copyright and Related Rights

Atty. Ferdinand Negre

Facts: ADA created the Code on Dental Procedures and Nomenclature which it published in 1969 on the Journal of the American Dental Association covered by a general copyright notice. It was revised frequently to adapt to changes and its 1991 & 1994 versions were also copyright registered. All dental procedures were classified into groups: each procedure receives a number, short description, and long description. Dental published a work entitled Universal Coding and Nomenclature including most of the numbering and short description from ADAs code. They claimed that they were entitled to reprint modified versions under an express or implied license, since they participated in the groups that drafted the Code, and fair use. It further claimed that since ADA distributed pamphlets containing the order versions without copyright notices it made the Code public domain forfeiting its copyright and that the subject matter was not copyrightable. District court ruled in favor of Delta saying that the taxonomy is not a copyrightable subject matter because it catalogs a field of knowledge. A comprehensive treatment cannot be selective in scope or arrangement and therefore cannot be original. Taxonomies are designed to be useful and if nothing remains after the useful is taken away if the primary function is removed from the form the work is devoid of even that modicum of creativity required. The Code is merely designed and used for business as nobody would read it for pleasure or aesthetic purposes. Plus, the fact that it was made by a committee makes it an oxymoron and can never be original. Issue: Whether taxonomy is copyrightable. Held: YES. Any original literary work may be copyrighted. The necessary degree of originality is low, and the work need not be aesthetically pleasing to be literary. A taxonomy is a way of describing items in a body of knowledge or practice; it is not a collection or compilation of bits and pieces of reality. The 1991 and 1994 versions of the Code may be recompilations of earlier editions, but the original Code is covered by Section 102 (a) as an original work of authorship, and its amendments by Section 106 (2) as derivative works. Delta argues that the Code is not copyrightable because it is a system. Section 102 (b) states that a copyright protection even of an original work does not cover any idea, procedure, process, system, method of operation, concept, principle, or discovery, regardless of the form in which it is described, explained, illustrated, or embodied in such work. The court ruled that the Code is not a system. The Code is a taxonomy, which may be put to many uses. These uses may be or include systems; the Code is not. (No other explanation was given) The Code was a how-to for dentists. So far as the ADA is concerned, any dentist, any insurer, anyone at all, may devise and use forms into which the Codes descriptions may be entered. Section 102 (b) precludes ADA from suing, for copyright infringement, a dentist whose office files record treatments using the Codes nomenclature. No field of practice has been or can be monopolized, given this constraint. Section 102 (b) permits Delta Dental to disseminate forms inviting dentists to use the ADAs Code when submitting bills to insurers. But it does not permit Delta to copy the Code itself, or make and distribute a derivative work based on the Code. (Ganito, parang cook book, di mo pwedeng kasuhan ung taong magluluto based on your book, pero pwede mong kasuhan ung taong gagaya nung book mo mismo). 18. Joaquin v. Drilon (1999) Facts: BJ Productions, Inc. (BJPI) is the holder of a Certificate of Copyright of a dating game show named Rhoda and Me. BJPI submitted to the National Library an addendum to its certificate of copyright specifying the shows format and style of presentation. While watching TV, Joaquin (President of BJPI) saw on RPN 9 an episode of Its a Date! which is produced by IXL Productions (IXL). He wrote a letter to the President of IXL, Zosa, informing him that BJPI had a copyright to Rhoda and Me and demanding that Its a Date be discontinued. Both shows have the same substance because 2 matches are made between a male and a female, both single, and the 2 couples are treated to a night or two of dining and/or dancing at the expense of the show. So, the major concepts are the same. 16 Roco 2013

Copyright and Related Rights

Atty. Ferdinand Negre

Zosa apologized and wanted to settle but IXL did not stop airing the show. Joaquin sent a second letter in which he reiterated his demand to discontinue Its a Date. Meanwhile, Zosa registered IXLs copyright to the first episode of Its a Date and he was given a certificate of Copyright by the National Library. Consequently, Joaquin sued Zosa. Sec. of Justice Drilon moved for the dismissal of the case. Zosa is contending that BJPIs copyright covers only a specific episode of Rhoda and Me and that the formats or concepts of dating game shows are NOT covered by copyright protection. Issue: (1) Whether format/mechanics of a show is entitled to copyright protection. (2) Whether master tapes need be presented. Held: (1) NO. The format of a show is not copyrightable. Section 2 of P.D. No. 49 enumerate the classes of work entitled to copyright protection. The format or mechanics of a television show is not included in the list of protected works in 2 of P.D. No. 49. For this reason, the protection afforded by the law cannot be extended to cover them. Copyright, in the strict sense of the term, is purely a statutory right. It is a new or independent right granted by the statute, and not simply a pre-existing right regulated by the statute. Being a statutory grant, the rights are only such as the statute confers, and may be obtained and enjoyed only with respect to the subjects and by the persons, and on terms and conditions specified in the statute. P.D. No. 49, 2, in enumerating what are subject to copyright, refers to FINISHED WORKS and not to concepts. The copyright does not extend to an idea, procedure, process, system, method of operation, concept, principle, or discovery, regardless of the form in which it is described, explained, illustrated, or embodied in such work. Mere description by words of the general format of the two dating game shows is insufficient; the presentation of the master videotape in evidence was indispensable to the determination of the existence of probable cause. As aptly observed by respondent Secretary of Justice: A television show includes more than mere words can describe because it involves a whole spectrum of visuals and effects, video and audio, such that no similarity or dissimilarity. (2) YES. Joaquin claim that Drilon gravely abused his discretion in ruling that the master videotape should have been presented in order to determine whether there was probable cause for copyright infringement. In the 20th Century Fox Film Corp vs. CA, it involved raids conducted on various videotape outlets allegedly selling or renting out pirated videotapes. The trial court found that the affidavits of NBI agents, given in support of the application for the search warrant, were insufficient without the master tape. The presentation of the master tapes of the copyrighted films, from which the pirated films were allegedly copied, was necessary for the validity of search warrants against those who have in their possession the pirated films. The essence of a copyright infringement is the similarity or at least substantial similarity of the purported pirated works to the copyrighted work. Hence, the applicant must present to the court the copyrighted films to compare them with the purchased evidence of the video tapes allegedly pirated to determine whether the latter is an unauthorized reproduction of the former. This linkage of the copyrighted films to the pirated films must be established to satisfy the requirements of probable cause. Mere allegations as to the existence of the copyrighted films cannot serve as basis for the issuance of a search warrant. This ruling was qualified in Columbia Pictures vs. CA: In fine, the supposed pronunciamento in said case regarding the necessity for the presentation of the master tapes of the copyrighted films for the validity of search warrants should at most be understood to merely serve as a guidepost in determining the existence of probable cause in copyright infringement cases where there is doubt as to the true nexus between the master tape and the pirated copies. An objective and careful reading of the decision in said case could lead to no other conclusion than that said directive was hardly intended to be a sweeping and inflexible requirement in all or similar copyright infringement cases. In the case at bar, during the preliminary investigation, Joaquin and Zosa presented written descriptions of the formats of their respective televisions shows, on the basis of which the investigating prosecutor ruled that the copyright does 17 Roco 2013

Copyright and Related Rights

Atty. Ferdinand Negre

not extend to the general concept or format of its dating game show. Accordingly, by the very nature of BJPIs copyright, the investigating prosecutor should have the opportunity to compare the videotapes of the two shows. 19. Pearl & Dean v. Shoemart (2003) Facts: Pearl & Dean Philippines (P&D), a below the line advertising company, entered into a contract with Shoemart (SM) to lease and install light boxes for advertising in their malls . Such light boxes were marketed under the trademarked name Poster Ads. Through their General Manager Rodolfo Z. Vergara, P&D entered into contracts covering SM Makati and SM Cubao, due to the fact that the original site of SM North EDSA had yet to be completed. SM signed the contract for SM Makati, but took action for the contract covering SM Cubao. Upon follow up on the SM Cubao contract, Vergara was informed that even the SM Makati contract was being rescinded. Metro Industrial Services (Metro) was the company that undertook to manufacture the light boxes for P&D. Metro, by themselves, later entered into a contract with SM. After this contract was done, SM engaged the services of EYD Rainbow Advertising Corporation for the same service. P&D caught wind of these transactions, and sued SM for copyright and trademark infringement, as well as unfair competition and damages. RTC ruled for P&D but CA reversed. Issue: Whether or not SM can be held liable for copyright, trademark and (by extension) patent infringement against P&D. Held: NO. P&D had a copyright certificate covering the light boxes under the name Advertising Display Units. However, such copyright was to protect only the technical drawings of the boxes, and not the actual boxes themselves. Under Sec 2 (O) of PD 49, prints, pictorial illustrations, advertising copies, labels, tags, and box wraps can be protected by copyright. Although P&Ds copyright certificate was entitled Advertising Display Units (which depicted the box-type electrical devices), its claim of copyright infringement cannot be sustained. Copyright, in the strict sense of the term, is purely a statutory right. Being a mere statutory grant, the rights are limited to what the statute confers. It may be obtained and enjoyed only with respect to the subjects and by the persons, and on terms and conditions specified in the statute. Accordingly, it can cover only the works falling within the statutory enumeration or description. P & D secured its copyright under the classification class O work. This being so, its copyright protection extended only to the technical drawings and not to the light box itself because the latter was not at all in the category of prints, pictorial illustrations, advertising copies, labels, tags and box wraps. State d otherwise, even as we find that P & D indeed owned a valid copyright, the same could have referred only to the technical drawings within the category of pictorial illustrations. It could not have possibly stretched out to include the underlying light box. The strict application of the laws enumeration in Section 2 prevents us from giving P&D even a little leeway, that is, even if its copyright certificate was entitled Advertising Display Units. What the law does not include, it excludes, and for the good reason: the light box was not a literary or artistic piece which could be copyrighted under the copyright law. And no less clearly, neither could the lack of statutory authority to make the light box copyrightable be remedied by the simplistic act of entitling the copyright certificate issued by the National Library as Advertising Display Units. The copyright extended to the technical drawings only, and the reproduction of the plans in the drawing would fall under patent infringement. It was admitted that the light box was neither a literary nor artistic work, but an engineering or marketing invention. P&D had no patent registration. However, the Court discussed the implications of having a patent registration arguendo.

18

Roco 2013

Copyright and Related Rights

Atty. Ferdinand Negre

It is only after an exhaustive examination by the patent office that a patent is issued. Such an In-depth investigation is required because in rewarding a useful invention, the rights and welfare of the community must be fairly dealt with and effectively guarded. To that end, the prerequisites to obtaining a patent are strictly observed and when a patent is issued, the limitations on its exercise are equally strictly enforced. To begin with, a genuine invention or discovery must be demonstrated lest in the constant demand for new appliances, the heavy hand of tribute be laid on each slight technological advance in art. There is no such scrutiny in the case of copyrights nor any notice published before its grant to the effect that a person is claiming the creation of a work. The law confers the copyright from the moment of creation and the copyright certificate is issued upon registration with the National Library of a sworn ex-parte claim of creation. Therefore, not having gone through the arduous examination for patents, the P&D cannot exclude others from the manufacture, sale or commercial use of the light boxes on the sole basis of its copyright certificate over the technical drawings. Stated otherwise, what P&D seeks is exclusivity without any opportunity for the patent office (IPO) to scrutinize the light boxs eligibility as a patentable invention. The irony here is that, had P&D secured a patent instead, its exclusivity would have been for 17 years only. But through the simplified procedure of copyright-registration with the National Library without undergoing the rigor of defending the patentability of its invention before the IPO and the public the P&D would be protected for 50 years. This situation could not have been the intention of the law. In the oft-cited case of Baker vs. Selden, the United States Supreme Court held that only the expression of an idea is protected by copyright, not the idea itself. In short, P&D opted for Copyright registration and not Patent registration. Therefore, the copyright protected the drawings or blueprints of the light boxes (expression) and not the actual construction of the boxes (idea). If a patent registration had been acquired, it would have been a different story altogether. 20. Ching v. Salinas (2005) Facts: Ching is the owner of Jeshicris Manufacturing Co., the maker and manufacturer of a Utility Model, described as Leaf Spring Eye Bushing for Automobile. Ching was issued a copyright registration and deposit for such product. Later, Ching asked help from the NBI for apprehension and prosecution of illegal manufacturers. The NBI was able to obtain search warrants from the RTC of Manila against Salinas Sr. and other officers of Wilaware Product Corporation, who were alleged of reproducing and distributing the models. Upon seizure of the goods, Salinas et al. moved to quash the warrants, saying that the models were not artistic or literary and that the models are spare parts of automobiles, which necessarily means they are not original; therefore, they are not copyrightable, but subject to patent. Ching opposed by saying that in the determination of probable cause in the issuance of a search warrant, the court cannot decide on the merits of copyrightability; furthermore, copyright registration has the presumption of validity. The RTC quashed the warrants, saying that a preliminary finding that a specific offense has been committed is needed to justify the issuance of a search warrant and therefore it can delve into the validity of the copyright. It then ruled that the registered models are not copyrightable. CA agreed, hence the petition to the SC. Issue: Whether the Utility Models are copyrightable Held: NO but they may be the subject of patent. The models (designs for spare parts in a car) are not literary or artistic works. They are not intellectual creations in the literary and artistic domain, or works of applied art. They are certainly not ornamental designs or one having decorative quality or value. The focus of copyright is the usefulness of the artistic design, and not its marketability. The central inquiry is whether the article is a work of art. Works for applied art include all original works that are intended to be or have been embodied in useful article regardless of factors such as mass production, commercial exploitation, and the potential 19 Roco 2013

Copyright and Related Rights

Atty. Ferdinand Negre

availability of design patent protection. While works of applied art, original intellectual, literary, and artistic works are copyrightable, useful articles and works of industrial design are not. A useful article may be copyrightable if and only if to the extent that such design incorporates pictorial, graphic, or sculptural features that can be identified separately from, and are capable of existing independently of, the utilitarian aspects of the article. There is no copyright protection for works of applied art or industrial design which have aesthetic or artistic features that cannot be identified separately from the utilitarian aspects of the article. In this case, Chings models are not works of applied art nor artistic works. They are utility models, useful articles, albeit with no artistic design or value. A utility model is a technical solution to a problem in any field of human activity which is new and industrially applicable. It may be, or may relate to, a product, a process, or an improvement of either. It is essentially an invention in the mechanical field. Being plain automotive spare parts that must conform to the original structural design of the components they seek to replace, the Leaf Spring Eye Bushing and Vehicle Bearing Cushion are not ornamental. They lack the decorative quality or value that must characterize authentic works of applied art. They are not even artistic creations with incidental utilitarian functions or works incorporated in a useful article. In actuality, the personal properties described in the search warrants are mechanical works, the principal function of which is utility sans any aesthetic embellishment. That the models may be the proper subject of a patent does not entitle Ching to the issuance of a search warrant for violation of copyright laws. Copyright and patent rights are completely distinct and separate from one another , and the protection afforded by one cannot be used interchangeable to cover items or works that exclusively pertain to the other. The dichotomy of protection for the aesthetic is not beauty and utility but art for the copyright and the invention of original and ornamental design for design patents. If the sole intrinsic function of an article is its utility, the fact that the work is unique and attractively shaped will not qualify it as a work of art. DERIVATIVE WORKS and COMPILATIONS 21. L. Batlin & Son, Inc. v. Snyder (1976) Facts: Uncle Sam mechanical banks (banks) have been on the American scene since 1886. This basic delightful design has long been in the public domain. These banks are found in collectors books and known to the average person. Snyder doing business as J.S.N.Y, obtained a registration of copyright on a plastic Uncle Sam bank in the category of Works of Art as sculpture. In his affidavit, Snyder said that he obtained a cast metal antique Uncle Sam Bank then he flew to Hong Kong to manufacture replicas as Bicentennial items. Snyder wanted his bank to be made of plastic and to be shorter than the case metal sample (11 inches to 9 inches). He also changed the shape of the bag and to include the umbrella in a one-piece mold. The plastic bank carried the legend CR Copyright J.S.N.Y. Batlin is also in the novelty business. He ordered 30 cartons of cast iron Uncle Sam mechanical banks from Taiwan. When he became aware of the existence of a plastic bank, which he considered an almost identical copy of the cast iron bank, Batlins trading company in Hong Kong procured a manufacturer and the president of Batlin ordered plastic copies also. Batlin was notified by the US Customs that the plastic banks were covered by Snyders copyright. The customs refused entry. So Batlin is instituting this case to declare Snyders copyright void and for damages for unfair competition and restraint of trade. The lower court granted Batlin a preliminary injunction. Snyder notes that there are differences between his copyrighted plastic model and the iron cast model. As for the bags shape, the latters was rough & fat while the formers was smooth & slim. Also, the eagle in the latter was holding arrows while the former was holding leaves. Essentially, there were differences in shape and texture which were not perceptible to the casual observer. 20 Roco 2013

Copyright and Related Rights Issue: Whether there was copyright infringement.

Atty. Ferdinand Negre

Held: NO. The court examined the copyrighted plastic model and the uncopyrighted model cast iron model. The Snyder bank is extremely similar to the cast iron bank, save in size and material with the only other differences, such as the shape of the satchel and the leaves in the eagles talons being by all appearances, minor. Similarities include, more importantly, the appearance and number of stripes on the trousers, buttons on the coat, and stars on the vest and hat, the attire and pose of Uncle Sam, the decor on his base and bag, the overall color scheme, the method of carpetbag opening, to name but a few. The plastic version reproduces the cast iron bank except that it proportionately reduces the height from approximately 11 inches to approximately 9 inches with trivial variations. There was no level of input which means that there was no great skill and originality. The test of originality is concededly one with a low threshold in that (a)ll that is needed is that the author contributed something more than a merely trivial variation, something recognizably his own. There were no elements of difference that amounted to significant alteration or that had any purpose other than the functional one of making a more suitable (and probably less expensive) figure in the plastic medium. The reproduction must contain an original contribution not present in the underlying work of art and be more than a mere copy. The mere reproduction of a work of art in a different medium should not constitute the required originality for the reason that no one can claim to have independently evolved any particular medium. Originality means that the work owes its creation to the author and this in turn means that the work must not consist of actual copying. The only case that appears to be an exception to this rule is the Hand of God case exact scale artistic reproduction of highly complicated statue made with great precision was original as requiring great skill and originality. To support a copyright, there must be at least some substantial variation, not merely a trivial variation such as might occur in the translation to a different medium. Nor can the requirement of originality be satisfied simply by the demonstration of physical skill or special training which, to be sure, Judge Metzner found was required for the production of the plastic molds that furnished the basis for appellants' plastic bank. A considerably higher degree of skill is required, true artistic skill, to make the reproduction copyrightable. Nor can the requirement of originality be satisfied simply by the demonstration of physical skill or special training. (remember Toyota case?) To extend copyrightability to minuscule variations would simply put a weapon for harassment in the hands of mischievous copiers intent on appropriating and monopolizing public domain work. DISSENT: In the instant case the author has contributed substantially more than a merely trivial variation. Any distinguishable variation of a prior work will constitute sufficient originality to support a copyright if such variation is the product of the author's independent efforts, and is more than merely trivial. In accord with the purposes of the copyright law to promote progress by encouraging individual effort through copyright protection, we should require only minimal variations to find copyrightability. The independent sculpting of the mold for the plastic bank and the aggregated differences in size and conformation of the figurine should satisfy this standard. It is hornbook that a new and original plan or combination of existing materials in the public domain is sufficiently original to come within the copyright protection. Originality sufficient for copyright protection exists if the author has introduced any element of novelty as contrasted with the material previously known to him. The most obvious differences between the two exhibits in this case are size and medium. While these factors alone may not be sufficient to render a work copyrightable, they surely may be considered along with the other variations. On the other hand, the author's reasons for making changes should be irrelevant to a determination of whether the differences are trivial.

21

Roco 2013

Copyright and Related Rights

Atty. Ferdinand Negre

22. Entertainment Research Group Inc. v. Genesis Creative Group, Inc. (1997) Facts: ERG designs and manufactures 3D inflatable costumes that are used in publicity events, such as shopping mall openings. The costumes are approximately 8ft tall and are worn by a person who remains inside the costume. Various companies purchase and use these costumes to promote their products. The costumes are based upon these companies' cartoon characters.(e.g. Pillsbury Doughboy, Geoffrey the Giraffe, CapnCrunch). ERG has manufactured and sold these walk-around inflatable costumes for the past 16 years. Over this time, ERG has developed techniques and designs that have resulted in high quality, attractive, durable and comfortable inflatable costumes. Up until the Spring of 1991, ERG was allegedly the only manufacturer utilizing these techniques. Genesis is in the business of marketing promotional and advertising devices to various companies. Aerostar is a manufacturer of hot air balloons, cold air inflatable products and cold air walk-around costumes. In 1989, ERG and Genesis began a business relationship. ERG's inflatable costumes were being sold to Genesis's customers. On September 18, 1990, ERG and Genesis entered into and signed a Confidential Disclosure Agreement. This agreement states that Genesis shall not disclose valuable and proprietary technical information and that Genesis shall not duplicate any of ERG's costumes. In August 1991, Genesis gave ERG written notice of the termination of all future business relations. Shortly thereafter, Genesis apparently entered into a formal business relationship with Aerostar to sell inflatable walk-around costumes manufactured by Aerostar to Genesis's customers. ERG claims that Genesis secretly entered into an agreement with Aerostar while the ERG-Genesis relationship was still alive and that Genesis supplied Aerostar with ERG-made costumes and ERG's proprietary information so that Aerostar could enter into the inflatable costume industry and so that Genesis could get itself a better deal through Aerostar. Thereafter, ERG sued Genesis and Aerostar for copyright infringement for the infringement of certain costumes. ERG alleges that Aerostar copied and infringed ERG's copyrights in the inflatable costumes that ERG designed and manufactured based on the underlying copyrighted characters. Issue: Whether Genesis is liable for copyright infringement of ERGs derivative works. Held: NO. To establish copyright infringement, the holder of the copyright must prove both valid ownership of the copyright and infringement of that copyright by the alleged infringer. It is undisputed that ERG's costumes are based upon 2D characters that are copyrighted and owned by the purchasers of the costumes. The Copyright Act defines a derivative work as a work based upon one or more pre-existing works, such as an art reproduction or any other form in which a work may be recast, transformed, or adapted. A work consisting of editorial revisions, annotations, elaborations, or other modifications which, as a whole, represent an original work of authorship is a derivative work. Given this definition, it is clear that ERG's costumes are derivative works for purposes of the Copyright Act. Nevertheless, the copyright protection afforded to derivative works is more limited than it is for original works of authorship. Copyright in a derivative work extends only to the material contributed by the author of such work, as distinguished from the pre-existing material employed in the work. To determine if said derivative work is copyrightable, ERG contends that the district court should have applied the Doran test. In Doran, a 3D inflatable representation of Santa Claus was original and copyrightable. Under said test, if the form of the derivative work and the form of the underlying work -3D, 2D, plastic, etc.-are sufficiently different, then the derivative work is original enough to be copyrightable. However, Doran is not applicable because it involved the copyrightability of a derivative work where the pre-existing work was taken from the public domain and not copyrighted itself. Here, ERG's costumes were based on pre-existing works that were copyrighted and owned by the ultimate purchasers. This difference is critical because in deciding whether to grant copyright protection to a derivative work, 22 Roco 2013

Copyright and Related Rights

Atty. Ferdinand Negre

courts must be concerned about the impact such a derivative copyright will have on the copyright privileges and rights of the owner of the underlying work. What must be applied is the DURHAM test. This test is 2-pronged: FIRST, to support a copyright, the original aspects of a derivative work must be more than trivial (originality). SECOND, the original aspects of a derivative work must reflect the degree to which it relies on pre-existing material and must not in any way affect the scope of any copyright protection in that pre-existing material. In other words, the copyright in such work is independent of, and does not affect or enlarge the scope, duration, ownership, or subsistence of, any copyright protection in the pre-existing material. Applying the first prong of Durham to the case, originality means little more than a prohibition of actual copying. No matter how poor artistically the author's addition, it is enough if it be his own. Any aspect of ERG's costumes that are purely functional, utilitarian or mechanical, will not be given any copyright protection. Moreover, any artistic aspects of ERG's costumes will also not receive copyright protection unless they can be identified separately from, and are capable of existing independently of, the utilitarian purpose of the costumes. Viewing the 3D costumes and the 2D drawings upon which they are based, it is immediately apparent that the costumes are not exact replicas of the 2D drawings. However, the 3D costumes were not considered as original because the added features were merely for utilitarian purposes. As for the second prong, ERG also failed to satisfy said element because, if ERG had copyrights for its costumes, any future licensee who was hired to manufacture costumes depicting these characters would likely face a strong copyright infringement suit from ERG. In other words, the rights of the holder of the underlying copyrighted characters would be affected. 23. Pickett v. Prince (2000) (Sequence: Princes symbol, Pickets guitar, Princes guitar) Facts: Prince Rogers Nelson (Prince) is a well-known singer who since 1992 referred to himself by an unpronounceable symbol serving as his trademark & a copyrighted work of visual art that licensees of Prince have embodied in various forms. He only registered the said symbol for copyright in 1997 but we know that copyright is obtained at the moment of creation and entitles the owner to protection, but registration is for purpose of instituting a suit for copyright infringement. Prince did not obtain the original copyright but he was the assignee that expressly granted him the right to sue even those infringement prior to the assignment. Pickett made a guitar in 1993 based from said symbol & allegedly showed it to Prince . He concedes that it is a derivative work within the contemplation of the law (derivative work is work based upon one or more preexisting works or any other form in which a work may be recast, transformed, or adapted.. representing an original work of authorship.) Shortly after that, Prince showed up in public playing a similar guitar. Suit was instituted in 1994 and in 1997. Prince filed a counterclaim for infringement of his symbol. During this time Prince had not yet copyrighted the symbol nor had it assigned in his favor. He anchored his claim on Warner Bros copyright when they produced his record and used the symbol in his album. The court ruled that WBs copyright was on the music not the symbol & even if they did, Prince had no standing to enforce someone elses copyright. Pickett claimed the right to copyright a work derivative from another persons copyright without permission and to sue that person for infringement by the persons own derivative work. He claimed his work had enough originality to entitle him to a copyright limited to the features that the derivative work adds. Issue: Whether or not Pickets suit for copyright infringement of his derivative work against Prince will prosper Held: NO. Although Picketts guitar is identical to Prince Symbol, the diff erence in appearance may well be due to nothing more than the functional difference because it is a musical instrument having strings, frets, etc. whereas the copyrighted symbol is a 2-d shape.

23

Roco 2013

Copyright and Related Rights

Atty. Ferdinand Negre

Copyright grants the owner of a copyright exclusive right to prepare derivative works based upon the copyrighted work. Pickett could not make a derivative work based on Princes symbol w/o authorization even if the guitar had a smidgeon of originality. A derivative work is bound to be very similar to the original and the law grants the right to make derivative works in the owner of the original to prevent endless series of infringement suits. Whether Princes guitar is a copy of his symbol or that of Picketts guitar is not a question to be answered in this litigati on because if anyone can just make derivative works based from copyrighted symbols there would be hundreds similar to Pickett charging others infringement. The right to make a derivative work does not authorize the maker to incorporate into it material that infringes someone elses copyright. By the definition of derivative it is difficult to see how a derivative work not made by the owner of the original could fail to infringe it. There can be no infringement by the owner because the law grants him exclusive right to make derivatives based on his copyrighted work. The only copyright that Pickett claims Prince infringed is a copyright that Pickett had no right to obtain since he did not have authority to make a derivative based on Princes symbol. Pickett could not copyright his guitar and his guitar infringes Princes copyright. 24. Feist Publications v. Rural Telephone (1991) 25. Tele-Direct Publications v. American Business Information (1997) Facts: This case was an appeal from the Trial Judges finding that copyright did not exist in the compilation of information contained in Yellow Pages directories. It was conceded that the Yellow Pages, taken as a whole and given the visual aspects of the pages and their arrangement, are protected by copyright. Teledirect Publications claimed copyright in the organization of the subscriber information (sub-directories) and in respect of the collection of additional data, such as facsimile numbers, trademarks and number of years in operation. The Trial Judge concluded that the appellant exercised only a minimal degree of skill, judgment or labor in its overall arrangement which was insufficient to support a claim of originality in the compilations so as to warrant copyright protection.

Prior to the 1993 North American Free Trade Agreement Implementation Act, compilations were protected only in so far as they could be characterized as literary works. In order to implement the North American Free Trade Agreement (NAFTA), the Copyright Act was substantially amended so that artistic, dramatic, literary and musical works included compilations thereof. Also, a definition of "compilation" was added to section 2 of the Copyright Act . The chain of events giving rise to the present claim both pre-dated and post-dated the amendments. Issues: (1) Whether the amendment modified the state of the law. (2) What was the correct approach for assessing the originality of the compilation (3) What role does skill, judgment, or labor play for qualification of copyright protection. Held: (1) NO. It only reinforced the law with respect to the fact that selection or arrangement of data results in a protected compilation only if the end result qualifies as an original intellectual creation. (2) The more correct approach to assessing the originality of a compilation where the work sought to be protected by copyright is a compilation of data which appears within a larger compilation of data is to begin the analysis with the fragment. The compilation at issue before the Trial Judge was not the directory as a whole (the main compilation), but fragments of that directory i.e. the in-column listings (the sub-compilation). (3) Essentially, for a compilation of data to be original, it must be a work that was independently created by the author and which displays at least a minimal degree of skill, judgment and labour in its overall selection or arrangement . 24 Roco 2013

Copyright and Related Rights

Atty. Ferdinand Negre

The threshold is low, but it does exist. If it were otherwise, all types of selections or arrangements would automatically qualify, for they all imply some degree of intellectual effort. Only those works which are original are protected. For compilation of data to be original, it must be a work that was independently created by the author, and display at least a minimal degree of skill, judgment and labor in its overall selection or arrangement. It is doubtful that considerable labour combined with a negligible degree of skill and judgment will be sufficient in most situations to make a compilation of data original. One of the purposes of the copyright legislation, historically, has been "to protect and reward the intellectual effort of the author in the work. The word author conveys a sense of creativity and ingenuity. The amount of labor is not a determinative source of originality. Th e Trial Judge correctly found that there was not a sufficient degree of originality in the sub-compilation. Intellectual property law must strike a balance between protecting the new products of inventive labour and allowing these to be freely available so as to form the basis for future progress. In the case at bar, it can hardly be said that the sub-compilation was a "new product of inventive labour" or that it amounted to an "intellectual creation" within the meaning of Article 1705 of NAFTA. The compilation of the in-column listings is of such an obvious and commonplace character as to be unworthy of copyright protection . Certain compilations of routine data are so mechanical as to be devoid of a creative element. 26. Desktop Marketing Systems v. Telstra (2002) Facts: Desktop Marketing (Desktop) took the whole of the Telstra white and yellow pages telephone directories including the classification headings, then rearranged and added information to the date recorded. Telstra sued for copyright infringement. The directories covered 55 geographic zones and the headings were an alphabetical list of cross-referenced category headings. The alleged infringing works comprise 3 CD-rom computer software products (Marketing Pro, Australian Phone Disk, and CD Phone Directory) produced by Desktop. Each CD-rom stores the Yellow and White pages listing data. There are marketing tools which enable the user to search the respective database in a variety of ways and to reproduce the results of searches. Telstra pleaded that Desktop produced 3 CD-rom products and that each of the products contained a reproduction of the whole or substantial part of the white pages, the yellow pages, and the headings. Desktop denied infringement on the ground that there was insufficient resemblance between the respective products. The Telstra products were alphabetical lists of subscribers. In effect, Desktop argued that it had taken fact not form. Telstras products were in column form, whilst the Desktop directories were in CD-rom form. Issue: (1) Whether Telstra is entitled to copyright in its White & Yellow Pages (2) Whether Desktop infringed Telstras copyright. Held: (1) YES. Compilation of raw facts may be a literary work. So far as compilations that consist entirely of existing material, that is, material not created by the compiler, are concerned, there is authority that originality may be found, and a claim of copyright may therefore subsist, by virtue of selection alone. Court also said that copyright can exist because the skill, judgment, and labour involved in putting that selection into written form, determines whether the compilation has sufficient originality to attract copyright. In considering whether a compilation that wholly consists of or that includes existing material is itself an original work, the question is whether the compilation, looked at as an entity, is original: it is not proper to dissect the work into its parts, and by determining that the individual parts lack originality, to deny originality to the whole work. The court held that copyright can be claimed in a compilation which: (1) has been produced as a result of the exercise of skill, judgment or knowledge in the selection, presentation or arrangement of the materials; or (2) has required the investment of a substantial amount of labour or expense to generate or collect the material included in it (the so-called sweat of the brow approach). 25 Roco 2013

Copyright and Related Rights

Atty. Ferdinand Negre

Literary Work includes a compilation. It is noteworthy that the noun work has been defined to mean, A thing, structure or result produced by the operation, action, or labor of a person or other agent and as a product of exertion, labor, or activity. In determining whether a work is original, that is, it originates from the author and is not copied, originality (like authorship), does not need to be novel, inventive or creative, whether of thought or in the expression. In a purely factual compilation, such as the white pages, the compiler must show that there was labour and expense required to collect the information and such labour and expense exceeded a minimum threshold. It is not the law that where there is only one way of expressing and arranging a factual compilation, the compilation cannot attract copyright protection. Note the important factor where one expression is inevitable, is whether the second expression was created independently of the first. Did the second author expend time and money to compile the data in the expression or did he or she merely utilise the first persons endeavour and labours to produce their own directory. Telstra had expended sufficient expense and labor to warrant the definition of originality. So Telstra had copyright. (2) YES. It was an infringement to reproduce, in whatever form, the details of all subscribers. Desktop had taken, not only quantitatively many pieces of factual information, it also took Telstras whole of universe certification . This term carries the meaning that there is no selection of the subscribers to be included. The universe of each regional directory is all the subscribers in the directory, except those with private numbers. What is the whole-of-universe certification? It is the assurance that the universe has been thoroughly explored and that all members of it have been captured. It gives value to the list. A compilation that does not capture all the members is not as valuable as one that can claim to have captured them all. The making of accessible whole-of-universe compilations is arguably to be encouraged by the giving of copyright protection on account of the industrious collection, verification, recording and assembly necessarily undertaken for the purpose. But the universe which Telstra identified was the names, addresses and telephone numbers (except private numbers) of all telephone subscribers within defined geographical regions. Telstra expended substantial labour in collecting, verifying, recording and assembling (albeit alphabetically) all this data so that it could issue a whole- of-universe listing. In one sense, Desktop did take Telstras form: it accurately reproduced all the individual names, addresses, telephone numbers and business descriptions, more or less as they individually appeared in the Directories. In another sense it did not do so: it did not reprodu ce Telstras alphabetical arrangement. Moreover, in one sense Desktop took ideas: it took the ideas denoted by names, addresses, telephone numbers and business descriptions. In another sense it did not do so: it did not take the idea of alphabetical arrangement. In the case of a factual compilation intended to be a work of utility, infringement must be tested by reference to the interest which copyright is intended to protect in the particular case. In the present case, that interest was the labour and expense of gathering together in the one place the details of all the members of a given universe - all the telephone subscribers in a region. What is the notion of substantial? It should not be understood as referring simply to quantity: it refers to the quality of the part taken. The most important indication of whether a part of a work is substantial is the originality of the part. It is important to inquire into the importance which the taken portion bears in relation to the work as a whole and to ask whether it was an essential or material part of the work. In this case, the substance of the information that has been taken from Telstras works (the directory portion of the directories and the headings that appear in the yellow pages directories and heading books) has been reproduced in the CD-ROMs. It must be remembered that copyright is not claimed for each particular entry, because copyright does not subsist in each individual recorded fact. It is claimed in the whole of the collected data, ordered in a particular way. As regards the directories, the significant recorded facts (name, address, telephone number, and the relevant type of business) are the same, or substantially the same, as they appear in Telstras works. While there are difference s, they are in the detail. For example, when displayed on a screen, the information from the CD-ROM does not appear as columns 26 Roco 2013

Copyright and Related Rights

Atty. Ferdinand Negre

on a page. But the information can be retrieved in alphabetical order (by postcode rather than region) and can be examined in much the same way as one would read a column on a page. The fact that the alphabetical listings are by postcode and not region, is not a material difference. Nor is the fact that portions of the advertisements are not reproduced. As regards the headings, it is true that they appear once only in each yellow pages directory and that the heading appears with each business entry in the CD-ROMs. This difference is immaterial. All the headings have been taken, as have all the listings beneath those headings. The appearance of the headings and the listings in the CD- ROMs is sufficiently similar to constitute a reproduction. The result is, and I so find, that Telstra has copyright in the white pages directories, yellow pages directories and headings books, the subject of this action, and Desktop has infringed that copyright. 27. Roth Greeting Cards v. United Card Co. (1970) Facts: Roth Greeting Cards (Roth) and United Card Company (United), both corporations, are engaged in the greeting card business. Roth brought this suit against United to recover damages and obtain injunctive relief for copyright infringement of 7 studio greeting cards. Roth's claim involves the production and distribution by United of 7 greeting cards which bear a remarkable resemblance to 7 of Roth's cards on which copyrights had been granted. Roth employed a writer to develop the textual material for its cards. On the other hand, during the period prior to the alleged infringements, United did not have any writers on its payroll . Most of its greeting cards came into fruition primarily through the activities of United's president, Mr. Koenig, and its VP, Edward Letwenko. Letwenko was unable to recall the origin of the ideas for most of United's cards. He speculated that the gags used may have come from plant personnel, persons in bars, friends at a party, Koenig, or someone else. He also stated that he visited greeting card stores and gift shows in order to observe what was going on in the greeting card business. Letwenko admitted that he may have seen the Roth cards during these visits or that the Roth cards may have been in his office prior to the time that he did his art work on the United cards. Issue: Whether there was copyright infringement of the greeting cards. Held: YES. In order to be copyrightable, the work must be the original work of the copyright claimant or of his predecessor in interest. But the originality necessary to support a copyright merely calls for independent creation, not novelty. United argues, and we agree, that there was substantial evidence that the textual matter of each card, considered apart from its arrangement on the cards and its association with artistic representations, was not original to Roth and therefore not copyrightable. However, proper analysis of the problem requires that all elements of each card, including text, arrangement of text, art work, and association between art work and text, be CONSIDERED AS A WHOLE. Thus, if United had copied only the textual materials, which were not independently copyrightable, United might have been able to do so with impunity. The Court held that the Roth cards are both original and copyrightable. Each of Roth's cards, considered as a whole, represents a tangible expression of an idea and that such expression was, in totality, created by Roth. As proper subjects of copyright, they are susceptible to infringement in violation of the Act. To constitute an infringement under the Act there must be substantial similarity, which must have been caused by defendant having copied the copyright holder's creation. It appears to us that in total concept and feel, the cards of United are the same as the copyrighted cards of Roth.

27

Roco 2013

Copyright and Related Rights

Atty. Ferdinand Negre

With the possible exception of one United card, the characters depicted in the art work, the mood they portrayed, the combination of art work conveying a particular mood with a particular message, and the arrangement of the words on the greeting card are substantially the same as in Roth's cards. In several instances the lettering is also very similar. It is true that each of United's cards employed artwork somewhat different from that used in the corresponding Roth cards. However, The test of infringement is whether the work is recognizable by an ordinary observer as having been taken from the copyrighted source. The remarkable similarity between the Roth and United cards in issue is apparent to even a casual observer. One Roth card has, on its front, a colored drawing of a cute moppet suppressing a smile and, on the inside, the words i wuv you. With the exception of minor variations in color and style, defendant's card is identical. Likewise, Roth's card entitled I miss you already, depicts a forlorn boy sitting on a curb weeping, with an inside message reading * * * and You Haven't even Left * * * is closely paralleled by United's card with the same caption, showing a forlorn and weeping man, and with the identical inside message. Since direct evidence of copying is rarely available, copying may be established by proof of access and substantial similarity. In the present case there was clear and uncontradicted testimony establishing United's access to the Roth cards. United brought Roth cards to its offices. It sent its employees out to gift shows and retail stores where the Roth cards were on display to observe what the competition was doing. These acts result to an inference that it was United's practice to look at the cards produced by other companies and make similar cards for sale under the United label. These circumstances, together with the marked similarity between the cards on which this suit was brought, with the possible exception of one card, convince us that each of United's cards in question, considered as combined compositions of art and text, were in fact copied from the Roth cards. DISSENT: I cannot, however, follow the logic of the majority in holding that the uncopyrightable words and the imitated but not copied art work constitutes such total composition as to be subject to protection under the copyright laws. The majority concludes that in the overall arrangement of the text, the art work and the association of the art work to the text, the cards were copyrightable and the copyright infringed. This conclusion, results in the whole becoming substantially greater than the sum total of its parts. The majority relied on Detective Comics, Inc. v. Bruns Publications, Inc. This case, as I read it, does not even intimate that the whole can exceed the sum total of its parts. It involved an intimation of the Superman image by a rival company. In finding infringement, the Superman court emphasized that So far as the pictorial representations and verbal descriptions of Superman are not a mere delineation of a benevolent Hercules, but embody an arrangement of incidents and literary expressions original with the author, they are proper subjects of copyright and susceptible of infringement because of the monopoly afforded by the Act.' Moreover, the Supermancase involved the creation of a character, a written dialogue and description which were clearly protected by copyright law in much the same way that a dialogue and description found in a novel is protected. The district court in the Superman case found that both the art work and the text materials were copyrightable and that each was infringed. In the case before us the text taken individually is not copyrightable and the images used were not copied. 28. Mason v. Montgomery Data, Inc. (MDI) (1992) Facts: Mason created and published 118 real estate ownership maps that cover all of Montgomery County. The maps, which display copyright notices, pictorially portray the location, size, and shape of surveys, land grants, tracts, and various topographical features within the county. Numbers and words on the maps identify deeds, abstract numbers, acreage, and owners of the tracts of land. He obtained the information from various sources like the US Geological Survey (USGS). Mason traced the survey and tract lines onto transparent overlays, enlarged clean USGS maps and the overlays, added names and other information to the overlays, and combined the maps and overlays to print the final maps. Mason registered the copyright for one of the original 118 maps in October 1968 . From 1970 to 1980, Mason revised the original maps and eventually published 115 new maps with copyright notices, for a total of 233 maps. 28 Roco 2013

Copyright and Related Rights

Atty. Ferdinand Negre

Landata purchased a set of Mason's maps and reorganized them by cutting and pasting them into 72 map sheets. This was for the purpose of creating a geographical indexing system for defendant MDI. On September 17, 1985, Landata asked Mason for permission to use his maps as part of its system, but Mason denied the request because Landata refused to pay a licensing fee. Landata then provided its products to MDI without Mason's permission. Using this system that Landata created, MDI was able to retrieve current ownership and other information on any tract. Annually from 1982 to1986, and again in 1989, Landata or MDI produced new, updated editions of the maps. Mason filed this suit in September 1988, claiming infringement of his copyrighted maps. Unfortunately, the court dismissed the case. The court claimed that to extend copyright protection to the Mason maps, which resulted from facts essentially in the public domain, would give the plaintiffs a monopoly over the facts. Essentially, the court used the MERGER DOCTRINE (merging of idea & expression) to rule against Mason. Issue: Whether Masons maps are protected by copyright. Held: YES. The Copyright Act extends copyright protection to original works of a uthorship fixed in any tangible medium of expression. The scope of that protection, however, is not unlimited. In no case does copyright protection for an original work of authorship extend to any idea, regardless of the form in which it is described, explained, illustrated, or embodied in such work. Thus, while a copyright bars others from copying an author's original expression of an idea, it does not bar them from using the idea itself . Others are free to utilize the idea so long as they do not plagiarize its expression. In some cases, however, it is so difficult to distinguish between an idea and its expression that the 2 are said to merge. Thus, when there is essentially only ONE WAY to express an idea, copying the expression will not be barred, since protecting the expression in such circumstances would confer a monopoly of the idea upon the copyright owner free of the conditions and limitations imposed by the patent law. By denying p rotection to an expression that is merged with its underlying idea, we prevent an author from monopolizing an idea merely by copyrighting a few expressions of it. To determine whether the MERGER DOCTRINE is applicable in any case, the court must focus on whether the idea is capable of various modes of expression. The court must first identify the idea that the work expresses, and then attempt to distinguish that idea from the author's expression of it. If found that the idea and its expression are inseparable, then the doctrine applies and the expression will not be protected. In all cases, the guiding consideration in drawing the line is the preservation of the balance between competition and protection reflected in the patent and copyright laws. Here, the record of the case contains copies of maps created by Mason's competitors that prove beyond dispute that the idea embodied in Mason's maps is capable of a variety of expressions. Although the competitors' maps and Mason's maps embody the same idea, they differ in the placement, size, and dimensions of numerous surveys, tracts, and other features. The record also contains affidavits in which licensed surveyors and experienced mapmakers explain that the differences between Mason's maps and those of his competitors are the natural result of each mapmaker's selection of sources, interpretation of those sources, discretion in reconciling inconsistencies among the sources, and skill and judgment in depicting the information. MDI argues that the merger doctrine applies in this case because drawing lines on a public map is the only way to depict the locations of surveys and boundary lines in Montgomery County. But, in this case, it is not in the methods available for depicting an object's location on a map, but in the ideas that the maps in the two cases embody. The IDEA here was to bring together the available information on boundaries, landmarks, and ownership, and to choose locations and an effective pictorial expression of those locations. That idea and its final expression are separated by Mason's efforts and creativity that are entitled to protection from competitors. The evidence in this case demonstrates that a mapmaker who desires to express the idea of depicting the location and ownership of property in Montgomery County in map form must select information from numerous sources, reconcile inconsistencies among those sources, and depict the information according to the mapmaker's skill and judgment . Extending protection to 29 Roco 2013

Copyright and Related Rights

Atty. Ferdinand Negre

that expression will not grant Mason a monopoly over the idea, because other mapmakers can express the same idea differently. The protection that each map receives extends only to its original expression, and neither the facts nor the idea embodied in the maps is protected. Originality does not require novelty, ingenuity, or aesthetic merit. Instead, originality means only that the work was independently created by the author (as opposed to copied from other works), and that it possesses at least some minimal degree of creativity. Mason's maps pass muster under Feist because Masons' selection, coordination, and arrangement of the information that he depicted are sufficiently creative to qualify his maps as original compilations of facts. D. Authorship SOLE AUTHORSHIP 29. Lindsay v. The Wrecked and Abandoned Vessel R.M.S Titanic (1999) BRIEFS: A person may be the author of a work even if he or she did not physically participate in the production of that work. Titanic had hired Lindsay to produce documentary footage detailing the salvaging of artifacts from the wreck of the Titanic. The filming was done by remote-controlled submarines. When a dispute arose as to Lindsays compensation for directing the filming, Titanic argued that Lindsay owned no copyright to the footage because he did not personally dive to the wreck to create it. Finding that Lindsay did own the copyright to the work, the court noted that the author of a work need not create that work in a literal, physical sense . Rather, Lindsay was the author because he exercised a high degree of control during the creative process such that the final product duplicates his conceptions and visions of what the film should look like. Facts: Lindsay is an independent documentary film maker. RMS Titanic, Inc (RMST) is a US Corp., Tullock their stockholder, president, and other companies including Discovery Channel (Discovery). RMST was awarded exclusive status as salvor-in-possession of the Titanic wreck site authorizing him to carry salvage operations but with the agreement that all artifacts recovered during said operations shall be maintained for historical verification, scientific education and public awareness. Lindsay had a contract with a British television company which filmed and directed Explorers of the Titanic , a chronicle of RMSTs 3rd salvage operation wherein Lindsay sailed with RMST for a month to make the documentary. During and after filming, he conceived a new film project using high illuminating lighting equipments. He discussed the idea with Tulloch wherein they both agreed to work together on the venture. Lindsay developed a comprehensive business plan for Titanic: A memorial tribute for approval of RMSTs board of directors. Lindsay even moved in to RMSTs office for said project wherein he was promised compensation, profit sharing, etc but the said contract was never executed. The problem here was that pre-production efforts such as creation of storyboards, drawings, specific camera angles, shooting sequences, etc reflecting Lindsays creative inspiration and force behind the concept for shooting had already been taught to RMSTs personnel and discussed. Huge underwater light towers were designed and personally constructed by Lindsay which was used for the film. 3-4 weeks serving as cinematographer, underwater videotaping, instructing the photographers for positioning and utilizing the light towers had also been done by Lindsay, all of which were later used in the film. He was never fully compensated for his services and RMST et al are now unlawfully profiting from the exploitation of the film project. Discovery Channel incorporated portions of Lindsays footage into different documentaries it aired. 13 causes of action were raised, 1 is copyright infringement. RMST claim that Lindsay cannot have copyright over said footage since he did not dive to the ship and did not himself actually photograph the wreckage. Issue: Whether or not Lindsays copyright over his video/photo compositions of the Titanic salvage operation was infringed.

30

Roco 2013

Copyright and Related Rights

Atty. Ferdinand Negre

Held: YES. The law provides that copyright ownership vests initially in the author of the work . The AUTHOR is the person who actually creates the work or the person who translates an idea into a fixed, tangible expression entitled to copyright protection. For purposes of film and photography, the author is the person who took the pictures but SC recognized that it may receive copyright protection when they represent original intellectual conceptions of the author such as in this case the storyboard, design, illuminating through the towers, angles of light and shoot, briefing with the cameramen on how to do it, daily review of the footage to weed out which will be included in the film all done by Lindsay existence of those facts of originality, intellectual production, thoughts and conception make them the proper subject of copyright protection. The fact that Lindsay did not literally shoot underwater does not defeat his claims of having authored the illuminated footage. He exercised a high degree of control over the film operation. JOINT AUTHORSHIP 30. Erickson v. Trinity Theatre Inc. (1994) Facts: Ms. Erickson was one of the founders of the trinity theatre and served it in several capacities, as playwright, artistic director, actress, play director, business manager, and member of the board of directors Ms. Erickson was paid by Trinity royalties for the performances of her plays. In 1988 she entered into a 2-year license agreement with Trinity as playwright and included the payment of royalties for the performance of her plays. As a playwright she prepared three plays for the company, namely: Much Ado, Time Machine, and Prairie Voices. (Much Ado: a compilation of different scenes and sonnets from the works of Shakespeare; other actors suggested parts to be included in it; Erickson who had the final decision of whether or not to include such suggestions. Time Machine: work began from the time Erickson was still teaching high school drama; such play involved the development of melodrama and improvisational scenes (ergo, no script); actors were given a theme which they would interpret themselves; said interpretations were compiled by Erickson into a script. Prairie Voices: intended by Ms. Erickson to be developed as a Trinity Production; originally intended to launch it as a collaborative effort in which the actors would contribute a story to the play) After the expiration of the licensing agreement, Trinity stopped paying Ms. Erickson royalties for the performance of her plays. They had a falling out. When Erickson left trinity in 1991 she obtained copyright registration for her plays and its videotapes. When Trinity continued to perform the plays, Erickson sued for copyright infringement. Erickson argued that she was the author, and therefore controlled the rights to the plays. Trinity argued that, since the actors made suggestions about the play to Erickson during production and rehearsals, Trinity was a co-author. The Court describes the setup Erickson had with the actors/ alleged co-authors during her stay as playwright. Issue: Whether Trinity, as joint author, can be liable to Erickson for copyright infringement. Held: YES. Trinity is not a joint author. Under copyright law, a joint work (aka one with multiple authors) can be licensed by any one of the authors, even over the objections of the others. Each author as co-owner has the right to use or to license the use of the work, subject to an accounting to the other co-owners for any profits. Copyright Act defines a joint work as a work prepared by 2 or more authors with the intention that their contributions be merged into inseparable or interdependent parts of a unitary whole. The problem with this definition is that neither the Act nor legislative history defines inseparable or interdependent. The touchstone here is the intention, at the time the writing is done, that the parts be absorbed or combined into an integrated unit. This ambiguity presents analytical problems in cases such as this one, in which the parties have collaborated in some sense but dispute whether there was a mutual intent to create a joint work. In resolving this ambiguity, we believe that it is 31 Roco 2013

Copyright and Related Rights

Atty. Ferdinand Negre

important to note, at the outset, that the statute itself requires that there be an intent to create a joint work. Therefore, reliance on collaboration alone, as Trinity suggests, would be incompatible with the clear statutory mandate. The Appellate Court considered two approaches for determining if someone was a co-author: The de minimis test, where everyone who contributed even a tiny bit to the whole is considered a co-author. (Nimmer) Here, for example, if 2 authors collaborate, with one contributing only uncopyrightable plot ideas and another incorporating those ideas into a completed literary expression, the 2 authors should be regarded as joint authors of the resulting work. This position is not supported because it is inconsistent with the fact that ideas and concepts, standing alone, should not be copyrightable. The copyrightable subject matter test, which says that a contributor is only a co-author if that person's contribution would be theoretically copyrightable by itself (Goldstein). Furthermore, the parties must have intended to be joint authors at the time the work was created. The Court found that the copyrightable subject matter test was the proper test to use. The Court found that the contributions of the actors was not enough to be independently copyrightable , therefore they could not be considered co-authors of the plays. Judgment of the district court is affirmed. 31. Aalmuhammed v. Lee (1999) BRIEFS: Where many individuals contribute creative elements to a work, the author of that work is the individual who superintends the creation of the work. Lee was the co-writer, co-producer, and director of a movie on the life of Malcolm X. While producing the movie, Lee hired Aalmuhammed as a consultant on the subject of Malcolm X. Aalmuhammeds made extensive script revision, directed actors on the set, created some new scenes with new characters, and edited parts of the film during post-production. While acknowledging that Aalmuhammeds contributions were extremely helpful, the court found that Aalmuhammed was not a coauthor of the movie because Lee retained artistic control. Facts: In 1991, Warner Brothers contracted with Spike Lee and others to make the movie Malcolm X, based on a book. Lee co-wrote the screenplay, directed, and co-produced the movie, which starred Denzel Washington as Malcolm X. Denzel asked Aalmuhammed to assist him in his preparations, as Aalmuhammed was a Malcolm X expert, having written, directed, and produced a Malcolm X documentary prior to the movie. Aalmuhammeds contributions: reviewed the script, suggested revisions and additions of entire scenes and dialogue, made sure the religious and historical aspects of Islam were portrayed accurately, acted as a translator (they worked in Egypt and he was the only one who knew Arabic), and edited parts of the movie. Aalmuhammed never had a written contract with Warner Bros., Spike Lee, or Lees production companies, but he expected to be paid for his services. He ultimately received $25,000 from Lee and $100,000 from Denzel. Before the movies release, Aalmuhammed asked for a writing credit as a co -writer of the film, but was turned down. Instead, upon the films release, he was credited only as an Islamic Technical Consultant. A few years later, Aalmuhammed applied for a copyright with the US Copyright Office, which was granted but with a reservation because it was in conflict with previous registrations. After the grant of copyright registration, Aalmuhammed filed a complaint against all involved in the movie for declaratory relief and accounting under the Copyright Act (and other actions for breach of contract, quantum meruit, unjust enrichment, and unfair competition). Aalmuhammed claimed that the movie was a joint work of which he was an author, thus making him a co-owner of the copyright of the entire movie. Issue: Whether Aalmuhammed is to be considered as co-author of Malcom X. Held: NO. Copyright Act does not define Author, but defines joint work as a work prepared by two or more authors with the intention that their contributions be merged into inseparable or interdependent parts of a unitary 32 Roco 2013

Copyright and Related Rights

Atty. Ferdinand Negre

whole. The statutory language establishes that for a work to be considered joint, there must be (1) a copyrightable work, (2) two or more authors, and (3) the authors must intend their contributions be merged into inseparable or interdependent parts of a unitary whole. A joint work, moreover, requires each author to make an independently copyrightable contribution to the disputed work . All persons involved intended that Aalmuhammed's contributions would be merged into interdependent parts of the movie as a unitary whole. Aalmuhammed maintains that he has shown a genuine issue of fact for each element of a joint work. But there is another element to a joint work. A joint work includes two or more authors. Aalmuhammed established that he contributed substantially to the film, but not that he was one of its authors. Authorship is required under the statutory definition of a joint work, and that authorship is not the same thing as making a valuable and copyrightable contribution. A contributor may be deemed the author of that expression, but only for the purpose of determining whether it is independently copyrightable or not. The word author is taken from the traditional activity of one person sitting at a desk with a pen and writing something for publication, which makes the application of the term author easy for one writing a novel. Who, in the absence of contract, can be considered an author of a movie? The word is traditionally used to mean the originator or the person who causes something to come into being. It can refer to the producer (who raises the money), the editor, the director (under the auteur theory), the screenplay writer, the star, or the cinematographer. In Burrow-Giles, the problem of defining author in new media was tackled. It was stated that author involves originating, making, producing, as the inventive or master mind, the thing which is to be protected; the man who really represents, creates, or gives effect to the idea, fancy, or imagination; he to whom anything owes its origin, originator, maker, one who completes a work of science or literature. Author requires more than a minimal creative or original contribution to the work. He is the person to whom the work owes its origin and who superintended the whole work, the master mind. A creative contribution does not suffice to establish authorship of the movie , as a contribution of independently copyrightable material to a work intended to be an inseparable whole does not suffice to establish authorship of a joint work. The Burrow-Giles definition of author, in the absence of a contract to the contrary, would generally limit authorship to someone at the top of the credits, sometimes the producer, director, star, or screenwriter someone who has artistic control. A person claiming to be an author of a joint work must prove that both parties intended each other to be joint authors. In the absence of contract, several factors may be considered, including who superintends the work by exercising control or the inventive or master mind who gives effect to the idea , the objective manifestations of shared intent to be co-authors, and the fact that the share of each in the works success cannot be appraised. Control in many cases will be the most important factor. The factors listed cannot be reduced to a rigid formula, because the creative relationships to which they apply vary too much. In this case, Aalmuhammed did not at any time have superintendence of the work; Warner Bros. and Spike Lee controlled it. Aalmuhammed lacked control of the work, and absence of control is strong evidence of the absence of co-authorship. WORKS MADE FOR HIRE 32. Community for Creative Non-Violence v. Reid (1989) BRIEFS: In deciding whether a particular work was made for hire, the reviewing court should first determine whether the creator of the work was an employee of the hiring party under common-law principles of agency. Once this determination has been made, the reviewing court should then analyze the works authorship under the appropriate paragraph of 101. The Community for Creative Non-Violence (CCNV) conceived of a crche-style structure depicting a homeless family huddled on the street. CCNV hired Reid to create the sculpture, and the parties engaged in preliminary discussions concerning the sculptures basic design and the material from which it would be made. While Reid was creating the sculpture, he employed assistants, who were paid by CCNV. A dispute then arose as to whether the sculpture should be taken on a touring exhibit. The parties disagreed as to who retained the final say over the disposition of the sculpture. CCNV contended that the sculpture was a work for hire, and Reid claimed the contrary. The court observed since sculpture does not fit within any of the 9 categories of specially ordered or commissioned works enumerated in the 33 Roco 2013

Copyright and Related Rights

Atty. Ferdinand Negre

law and there was no written agreement between the parties as to Reids status, [t]he dispositive inquiry in this case [ ] is whether [the sculpture] is a work prepared by an employee within the scope of his or her employment. After reviewing the factors tending to establish an employeremployee relationship, the court found that Reid was not an employee of CCNV. The sculpture was therefore not a work for hire made for CCNV, though the court observed that CCNV might qualify as a co-author. Facts: CCNV decided to participate in the annual Christmas time Pageant of Peace in Washington, D.C., by sponsoring a display to dramatize the plight of the homeless. Snyder (a CCNV trustee) and fellow CCNV members conceived the idea for the nature of the display: a sculpture of a modern nativity scene. They also settled upon a title for the work Third World Americaand a legend for the pedestal: and still there is no room at the inn. Snyder was referred to James Earl Reid, a sculptor. Both parties agreed that the project would cost no more than $15,000, not including Reid's services, which he offered to donate. The parties did not sign a written agreement. Neither party mentioned copyright. At Snyder's request, Reid sent CCNV a sketch of a proposed sculpture. Reid testified that Snyder asked for the sketch to use in raising funds for the sculpture. Snyder testified that it was also for his approval. Reid worked exclusively on the statue, assisted at various times by a dozen different people who were paid with funds provided in installments by CCNV. On a number of occasions, CCNV members visited Reid to check on his progress and to coordinate CCNV's construction of the base. Days after the agreed upon date, Reid delivered the completed statue to Washington. The statue remained on display for a month. In late January 1986, CCNV members returned it to Reid's studio for minor repairs. Several weeks later, Snyder began making plans to take the statue on a tour of several cities to raise money for the homeless . Reid objected, contending that the material was not strong enough to withstand the ambitious itinerary. He urged CCNV to cast the statue in bronze or to create a master mold. Snyder declined to spend more of CCNV's money on the project. In March 1986, Snyder asked Reid to return the sculpture. Reid refused. He then filed a certificate of copyright registration for the sculpture in his name and announced plans to take the sculpture on a more modest tour than the one CCNV had proposed. Snyder, acting in his capacity as CCNV's trustee, immediately filed a competing certificate of copyright registration. Snyder and CCNV then commenced this action against Reid and his photographer for determination of copyright ownership. District Court declared that it was a work made for hire under 101 of the Copyright Act and that Snyder, as trustee for CCNV, was the exclusive owner of the copyright in the sculpture. CA reversed. Issue: Whether the work falls under work-for-hire such that CCNV can be deemed the author of the work for copyright purposes. Held: NO. The sculpture was not a work made for hire. CCNV was not the author of the sculpture. Therefore, CCNV is not entitled to the copyright. In cases of works made for hire, the employer or other person for whom the work was prepared is considered the author and owns the copyright, unless there is a written agreement to the contrary. Classifying a work as made for hire determines not only the initial ownership of its copyright, but also the copyright's duration, and the owners' renewal rights, termination rights, and right to import certain goods bearing the copyright. Under the law, a work is for hire under any of the ff: (1) work prepared by an employee within the scope of employment; or (2) a work specially ordered or commissioned. To determine whether a work is for hire under the Act, a court first should ascertain, using principles of general common law of agency, whether the work was prepared by an employee or an independent contractor . After making this determination, the court can apply the appropriate subsection. The hiring party's right to control the product simply is not determinative . In determining whether a hired party is an employee under the general common law of agency, we consider the hiring party's right to control the manner and means by which the product is accomplished. Factors relevant to this inquiry are: (1) the skill required; (2) source of the 34 Roco 2013

Copyright and Related Rights

Atty. Ferdinand Negre

instrumentalities and tools; (3) the location of the work; (4) the duration of the relationship between parties; (5) whether the hiring party has the right to assign additional projects to the hired party; (6) the extent of the hired partys discretion over when and how long to work; (7) the method of payment; (8) the hired partys role in hiring and paying assistants; (9) whether the work is part of the regular business of the hiring party; (10) whether the hiring party is in business; (11) the provision of employee benefits; and (12) the tax treatment of the hired party. Since these factors were absent, the Court held that Reid was not an employee of CCNV but an independent contractor. True, CCNV members directed enough of Reid's work to ensure that he produced a sculpture that met their specifications. But the extent of control the hiring party exercises over the details of the product is not dispositive. All the other circumstances weigh heavily against finding an employment relationship. Reid is a sculptor, a skilled occupation. Reid supplied his own tools. He worked in his own studio in Baltimore, making daily supervision of his activities from Washington practicably impossible. Reid was retained for less than 2 months, a relatively short period of time. During and after this time, CCNV had no right to assign additional projects to Reid. Apart from the deadline for completing the sculpture, Reid had absolute freedom to decide when and how long to work. CCNV paid Reid $15,000, a sum dependent on completion of a specific job, a method by which independent contractors are often compensated. Also, under the law, Sculpture does not fit within any of the categories of specially ordered or commissioned works. CCNV nevertheless may be a joint author of the sculpture if, on remand, the District Court determines that CCNV and Reid prepared the work with the intention that their contributions be merged into inseparable or interdependent parts of a unitary whole. In that case, CCNV and Reid would be co-owners of the copyright in the work. 33. Aymes v. Bonelli (1992) Facts: Bonelli is the president and CEO of Island. Island operated a chain of retail stores selling swimming pools and related supplies. In May 1980, Aymes, a Cornell graduate, was hired by Bonelli to work as a computer programmer where he worked until 1982. During that period, Aymes created a series of programs called CSALIB under the general direction of Bonelli who was not a programmer. The program was used by Island to maintain records of cash receipts, physical inventory, sales figures, purchase orders, merchandise transfers, and price changes. There was no written agreement between Bonelli and Aymes assigning ownership or copyright of CSALIB. Aymes does contend, however, that Bonelli made him an oral promise that CSALIB would only be used at one computer in one Island office. Aymes did most of his programming at the Island office, where he had access to Island's computer hardware. He generally worked alone, without assistants or co-workers, and enjoyed considerable autonomy in creating CSALIB. This autonomy was restricted only by Bonelli who directed and instructed Aymes on what he wanted from the program. Although Aymes worked semi-regular hours, he was not always paid by the hour and on occasion presented his bills to Bonelli as invoices. At times, Aymes would be paid by the project and given bonuses for finishing the project on time. He never received any health or other insurance benefits from Island. Also, Island never paid an employer's percentage of Aymes's payroll taxes and never withheld any of his salary for federal or state taxes. In fact, Aymes was given an Internal Revenue Service Non-Employee Compensation form instead of the standard employee. Aymes left Island in September 1982 when Bonelli unilaterally decided to cut Aymes's hours. At the time Aymes left, Island owed him $14,560 in wages. Aymes also requested payment for multisite use of CSALIB. When he became persistent in his demands for compensation, however, Bonelli insisted that he sign a release for his rights to CSALIB in order to receive the back earnings. Aymes refused to sign and was not paid. As such, he registered CSALIB in his own name with the US Copyright Office. Then, he filed a complaint against Bonelli for copyright infringement. Issue: Whether Bonelli infringed on Aymes copyright over the program.

35

Roco 2013

Copyright and Related Rights

Atty. Ferdinand Negre

Held: YES. The Act defines a work made for hire as: (1) a work prepared by an employee within the scope of his or her employment; or (2) a work specially ordered or commissioned for use if the parties expressly agree in a written instrument signed by them that the work shall be considered a work made for hire. Aymes and Bonelli never signed a written agreement assigning ownership rights in CSALIB. We must therefore consider whether the program was a work prepared by Aymes as an employee within the scope of his employment. In determining whether a hired party is an employee under the general common law of agency, we consider the hiring party's right to control the manner and means by which the product is accomplished . Among the other factors relevant to this inquiry are the skills required, the source of the instrumentalities and tools, the location of the work; the duration of the relationship between the parties, whether the hiring party has the right to assign additional projects to the hired party; the extent of the hired party's discretion over when and how long to work; the method of payment; the hired party's role in hiring and paying assistants; whether the work is part of the regular business of the hiring party; whether the hiring party is in business; the provision of employee benefits, and the tax treatment of the hired party. RIGHT TO CONTROL: It is clear from the record that Bonelli and Island had the right to control the manner in which CSALIB was created. Aymes disputed Bonelli's purported skill at programming, but even without such knowledge Bonelli was capable of directing Aymes on CSALIB's necessary function. Aymes was not working entirely alone. He received significant input from Bonelli in programming CSALIB , and worked under programming limitations placed by Bonelli. Consequently, this factor weighs heavily in favor of finding that Aymes was an employee. LEVEL of SKILL: Aymes's work required far more than merely transcribing Bonelli's instructions. Rather, his programming demanded that he use skills developed while a graduate student at Cornell and through his experience working at a family run company. Other courts that have addressed the level of skill necessary to indicate that a party is an independent contractor have held architects, photographers, graphic artists, drafters, and indeed computer programmers to be highly-skilled independent contractors. As such, this weighs heavily in Aymes favor. EMPLOYEE BENEFITS & TAX TREATMENT: Aymes was not provided with health, unemployment, or life insurance benefits. Similarly, Island did not pay a share of Aymes's social security taxes and did not withhold federal or state income taxes. The failure of Island to extend Aymes any employment benefits or to pay any of his payroll taxes is highly indicative that Aymes was considered an outside independent contractor by Island. These two factors are given even greater weight because they are undisputed in this case. During the ten years in which this case has been litigated, all the other issues have been hotly contested. But for purposes of benefits and taxes, Island definitely and unequivocably chose not to treat Aymes as an employee. RIGHT TO ASSIGN OTHER PROJECTS: Bonelli had the right to and did assign Aymes other projects in addition to the creation of CSALIB. This is fairly strong evidence that Aymes was an employee, since independent contractors are typically hired only for particular projects. However, this factor carries less weight than those evaluated above, because the delegation of additional projects to Aymes is not inconsistent with the idea that he was Island's independent trouble shooter who might be asked to intervene as computer problems arose. Examining the factors for each side in terms of their importance, we conclude that the only major factor strongly supporting Island is that it directed the creation of the program. Supporting Aymes's argument that he was an independent contractor, however, are several important factors his skill, and the tax and benefit factors that outweigh the elements supporting Island. On balance, application of the Reid test requires that we find Aymes to be an independent contractor when he was creating CSALIB for Island. Consequently, we hold that CSALIB is not a work for hire. Aymes therefore owns the copyright as author of the program. 34. Roeslin v. District of Columbia (1995) Facts: Roeslin was employed by Department of Employment Services of District of Columbia as a labor economist. He had no programming skills at that time nor was he hired for such skill. His tasks involved: (1) Improve employer 36 Roco 2013

Copyright and Related Rights

Atty. Ferdinand Negre

response rate to CES survey; (2) Expand CES sample size; (3) Develop industry and occupational employment projections. DOES used a manual system in collating these data wherein employees collect information from returned CES survey and records them in office record cards. District anticipated future development of the Automated Current Employment Statistics (ACES). Roeslin used computers to do his work but it did not involve programming and he assisted also in transferring data through the use of computer software applications existing. During a CES conference, Roeslin was motivated to create the DC-790 and he informed his supervisor that this is possible using a PC based system to create the surveys. His supervisor discouraged him saying its not feasible nor desirable and he would be too busy with other duties and they would eventually transition to ACES. Roeslin still decided to pursue this project on his own time, buying his own PC, softwares and books to learn how to create the system. He wanted to prove it could be done and create job opportunities for himself but his supervisor told him that this program would become public domain which he meant as something that would belong to District but Roeslin interpreted as something not owned by anybody and free for all. During the entire development no supervision, assistance, compensation was offered by District to Roeslin. He did it all on his own. He was eventually able to create it and tried it in their office. It was a success and everybody wanted to use it so he made a manual and conducted demos to promote the said program. He received good feedback and was up for promotion. All the while he relied on his supervisors statement that it was public domain, but later on found out that District is claiming proprietary interest over his program which prompted him to confront his supervisor saying that he should be given credit and if he be promoted and recognized he would allow District to use it freely. He registered DC-790 and a copyright notice was placed on the initial screen of the said program. He informed District and the Mayors office of his claim and demanded that they stop using it. He was just ignored by district who continued to use it, installed it in many computers and never applied for copyright under its name. He informed District that he will not render any modification and programming connected to his copyright claim, he was transferred to a computer programmer analyst but never worked in connection with DC-790 even when it became operational due to employee error. Eventually ACES was implemented and paid system cost composed of programmers salaries and processing charges which an employee of DOES admitted not being paid for the use of DC-790. Issue: Whether or not Roeslin is the author of DC-790. Held: YES. Roeslin is the owner because the program was not made for hire. When a plaintiff has a copyright registration, burden is on defendant to prove that the work is a work made for hire. To prove copyrighted work is a work made for hire, employer must prove that employees conduct in making work is within scope of employment, occurs substantially within authorized time and space limits, actuated for purpose of serving the employer. Developing computer software was not the kind of work that labor economist was employed to perform, for purpose of determining whether program written by employee to receive, process, and transmit survey results was a work made for hire; there was no reference in his job description to computer programming and he was not hired for that skill. The court held that to determine whether a particular conduct was incidental to employees employment, they may consider factors as whether activity was the type of activities commonly done by persons employed in the same position. It was shown that labor economist have nothing to do with software programming. Further, the entire system was developed using Roeslins own resources and funds and during his own time outside work (3000 hrs). He was never motivated to create computer program for purpose of serving his employer but to create for himself a job opportunity and prove it could be done. 37 Roco 2013

Copyright and Related Rights

Atty. Ferdinand Negre

His failure to assert a proprietary interest in the computer program did not defeat his action for copyright infringement against employer where his failure was due to employers representation that employee would have no proprietary interest in the program.

38

Roco 2013

You might also like